Forumul Scientia

Geologie, mediu şi ştiinţe planetare => Geologie si stiinte planetare => Subiect creat de: tavy din Ianuarie 31, 2011, 08:32:39 PM

Titlu: Măsurarea temperaturii atmosferice și punerea în evidență a schimbărilor climatice
Scris de: tavy din Ianuarie 31, 2011, 08:32:39 PM
Citat din: Adi din Ianuarie 31, 2011, 08:16:38 PM
ci doar sunt curios daca publicul roman chiar stie ca incalzirea globala e realitate
Să înțeleg că tu susții că avem de-a face cu o încălzire globală? Cum ai determinat asta?

Citat din: mircea_p din Ianuarie 31, 2011, 08:18:17 PM
Sondajul e pe site-ul RADO, cel putin daca urmezi linkul dat de OP.
Acum am găsit site-ul. Este cel puțin ciudat că acest ,,Observator Atmosferic Român" nu are un site în limba română. Am impresia că este doar un alt ,,institut" care sifonează fonduri cu minciuna încălzirii globale cauzată de activitatea umană.

CitatToday News: January,  2011
Global warming is already affecting the earth in a variety of ways. Some researchers indicates that many tree species might become extinct due to climate change if no action is taken in time.
Ăștia nici nu au aflat că nu se mai discută despre încălzire globală, de când modelele climatice care previzionau această încălzire au fost falsificate, acum sunt la modă ,,schimbarile climatice". Cu schimbările climatice este mai greu să o dai în bară, dacă previzionezi ,,schimbări climatice" ai șanse minime să greșești pentru că până acum nu știu să se cunoască o perioadă în istorie când să nu fi avut loc schimbări climatice.
Titlu: Măsurarea temperaturii atmosferice și punerea în evidență a schimbărilor climatice
Scris de: AlexandruLazar din Ianuarie 31, 2011, 08:46:56 PM
După engleza de baltă de pe site, cu siguranţă e al unui institut român. În rest -- doar un comentariu, eu nu ţin minte să se fi făcut vreo discuţie serioasă despre încălzirea globală pe baza unor modele teoretice care să prevadă asta, ci pe baza unor măsurători care indică destul de clar creşterea temperaturii medii anuale şi pe perioade de cinci ani în toate regiunile Terrei. Nu ştiu a ce sună asta pentru alţii, da' pentru mine sună a încălzire.
Titlu: Măsurarea temperaturii atmosferice și punerea în evidență a schimbărilor climatice
Scris de: Adi din Ianuarie 31, 2011, 08:51:36 PM
Citat din: mircea_p din Ianuarie 31, 2011, 08:18:17 PM
Sondajul e pe site-ul RADO, cel putin daca urmezi linkul dat de OP.
E cam greu sa nu-l gasesti.

El se referea la altceva. Ca daca cauti pe google dupa "Observatorul Atmosferic Roman" pe prima pagina nu gasesti site-ul RADO, ceea ce arata ca au dus o munca de aproape zero pentru optimizarea siteului pentru google, sau macar de a il promova cu linkuri spre el de la mai multe siteuri, sub numele de "Observatorul Atmosferic Roman". Aici sunt complet de acord cu el. Trebuie ca laboratorul sa faca ca siteul lui sa poata fi gasit pe google.
Titlu: Măsurarea temperaturii atmosferice și punerea în evidență a schimbărilor climatice
Scris de: Adi din Ianuarie 31, 2011, 08:54:35 PM
Citat din: tavy din Ianuarie 31, 2011, 08:32:39 PM
Citat din: Adi din Ianuarie 31, 2011, 08:16:38 PM
ci doar sunt curios daca publicul roman chiar stie ca incalzirea globala e realitate
Să înțeleg că tu susții că avem de-a face cu o încălzire globală? Cum ai determinat asta?

Am facut o greseala de ortografie. Voiam sa spun ca "ei sunt curiosi". Si mi se pare foarte corect sa fie curiosi ce crede publicul roman despre incalzirea globala.

Citat
CitatToday News: January,  2011
Global warming is already affecting the earth in a variety of ways. Some researchers indicates that many tree species might become extinct due to climate change if no action is taken in time.
Ăștia nici nu au aflat că nu se mai discută despre încălzire globală, de când modelele climatice care previzionau această încălzire au fost falsificate, acum sunt la modă ,,schimbarile climatice". Cu schimbările climatice este mai greu să o dai în bară, dacă previzionezi ,,schimbări climatice" ai șanse minime să greșești pentru că până acum nu știu să se cunoască o perioadă în istorie când să nu fi avut loc schimbări climatice.

Imi pare rau, dar nu exista nicio dezbatere despre incalzirea globala la nivel de oameni de stiinta. A fost arata ca exista. Eventual e dezbatere daca ea e produsa de oaemni in proportie de 90% sau doar de 1%, dar incalzirea globala e reala. Doar trebuei sa te uiti la graficul temperaturii anuale si globale de la 1770 incoace.

Nu exista nicio dovada a falsificarii datelor. Daca te referi la scandalul de anul trecut de la East Anglia University, ei bine, e doar scandal fara dovezi, si in plus dovezile incalzirii globale nu erau doar de la acele studii, ci de la alte studii independente, gen NASA ...

Dar independent de asta, ei nu au spus ca exista incalzire globala, ci doar au intrebat pe oameni daca ei cred ca exista sau nu. Deci au facut complet corect.
Titlu: Măsurarea temperaturii atmosferice și punerea în evidență a schimbărilor climatice
Scris de: Adi din Ianuarie 31, 2011, 08:56:31 PM
Citat din: AlexandruLazar din Ianuarie 31, 2011, 08:46:56 PM
După engleza de baltă de pe site, cu siguranţă e al unui institut român. În rest -- doar un comentariu, eu nu ţin minte să se fi făcut vreo discuţie serioasă despre încălzirea globală pe baza unor modele teoretice care să prevadă asta, ci pe baza unor măsurători care indică destul de clar creşterea temperaturii medii anuale şi pe perioade de cinci ani în toate regiunile Terrei. Nu ştiu a ce sună asta pentru alţii, da' pentru mine sună a încălzire.

Complet de acord. Incalzirea e masurata, e clara. Apoi modele pot doar explica de ce s-a incalzit si sa estimeze cat se va incalzi in viitor. Acolo, da, exista dezbatere. Dar ca s-a incalzit de la 1770 incoace, aia e clar.
Titlu: Măsurarea temperaturii atmosferice și punerea în evidență a schimbărilor climatice
Scris de: tavy din Ianuarie 31, 2011, 09:02:09 PM
Citat din: AlexandruLazar din Ianuarie 31, 2011, 08:46:56 PM
După engleza de baltă de pe site, cu siguranţă e al unui institut român. În rest -- doar un comentariu, eu nu ţin minte să se fi făcut vreo discuţie serioasă despre încălzirea globală pe baza unor modele teoretice care să prevadă asta, ci pe baza unor măsurători care indică destul de clar creşterea temperaturii medii anuale şi pe perioade de cinci ani în toate regiunile Terrei. Nu ştiu a ce sună asta pentru alţii, da' pentru mine sună a încălzire.
Nu știu ce studii ai, poate nu ai cunoștințe de fizică dar temperatura este o mărime fizică intensivă, spre deosebire de mărimile fizice extensive media marimilor fizice intensive nu are nici o semnificație fizică. Cu alte cuvinte orice studiu care vorbește despre ,,temperatura medie" nu este un studiu stiințific. Oricum încălzirea nu este obligatoriu legată de creșterea temperaturii, în realitate un sistem se încălzește atunci când căldura primită este mai mare decât căldura cedata. Poți spre exemplu să încălzești (cvasistatic) un vas cu amestec de apă și ghiață la echilibru și temperatura acestuia nu se va modifica până nu se va topi toată ghiața.

Titlu: Măsurarea temperaturii atmosferice și punerea în evidență a schimbărilor climatice
Scris de: razvanc87 din Ianuarie 31, 2011, 09:04:09 PM
Bun, am inteles nemultumirile voastre legate de acest chestionar. Si in mare parte sunt de acord cu voi, dar nu sunt de acord cu idei precum Institutul National de C-D pentru Optoelectronica nu ar avea un departament care studiaza mediul, din moment ce eu fac parte din acel departament si de-asta am si postat chestionarul pe forumurile mai populare pe care le stiu.

Este in interesul nostru sa aflam daca menatlitatea Romaniei s-a schimbat in vreu fel anume, si aici ma refer mai ales la modul in care oamenii sunt informati si se informeaza despre ce se intampla in ograda lor. Cei din Romania care ati v-ati uitat in perioada eruptiei vulcanice din Islanda poate isi amintesc niste articole despre INOE care au fost la Realitatea TV, ProTV, TVR1 si ca Departamentul LAST din care fac parte s-a ciondanit putin cu ANM pentru ca cei de-acolo lucrau pe modelari, iar noi aveam masuratori. In fine, daca nu va mai amintiti poate gasiti in arhivele acelor site-uri pentru cei ce nu cred.

Cat despre site, el este facut de sefa mea si nu este unul foarte bun, initial trebuia sa-l fac eu, dar am fost prea... extravagant in alegerile pe care le-am facut, nu i-au placut asa ca am lasat-o sa se descurce singura, fonduri pentru asta nu am avut... E normal sa nu apara ca prima optiune pe Google din moment ce este facut de cineva care a folosit nu stiu ce soft de facut site-uri, dar informatiile de pe el sunt cat se poate de reale, indiferent de modul de vizualizare ale lor.

Ca sa vedeti comparatie intrati pe http://rado.inoe.ro/Quicklooks
Acesta este un site facut de mine, tot pentru RADO dupa cum se vede si in nume, care este pe un domeniu al INOE dupa cum se vede si din nume. Si sa zici ca un Institut National e de balta... asta denota dezinformare, din moment ce se afla pe cea mai mare platforma de institute din Romania si anume Magurele.

Edit:
Apropos, am uitat sa spun asta:
- ca test, pentru cei ce sunt interesati, pe site-ul cu Quicklooks, daca alegeti in ordine:
(1) Stations - INOE etc.
(2) Categories - Special Events
(3) Parameters - Volcanic Ash
Submit

O sa gasiti o galerie cu ploturi si imagini produse de un LIDAR ce au fost masurate de catre departamentul nostru in perioada cu cenusa vulcanica, iar din acele poze LIDAR se observa clar straturile atmosferice, care mai apoi s-au determinat a fi intr-adevar cenusa vulcanica din Islanda.
Titlu: Măsurarea temperaturii atmosferice și punerea în evidență a schimbărilor climatice
Scris de: mircea_p din Ianuarie 31, 2011, 09:10:13 PM
Citat din: tavy din Ianuarie 31, 2011, 09:02:09 PM
Nu știu ce studii ai, poate nu ai cunoștințe de fizică dar temperatura este o mărime fizică intensivă, spre deosebire de mărimile fizice extensive media marimilor fizice intensive nu are nici o semnificație fizică. Cu alte cuvinte orice studiu care vorbește despre ,,temperatura medie" nu este un studiu stiințific. Oricum încălzirea nu este obligatoriu legată de creșterea temperaturii, în realitate un sistem se încălzește atunci când căldura primită este mai mare decât căldura cedata. Poți spre exemplu să încălzești (cvasistatic) un vas cu amestec de apă și ghiață la echilibru și temperatura acestuia nu se va modifica până nu se va topi toată ghiața.
Esti de acord cel putin ca vara e mai cald decat iarna?
Dupa ce spui aici s-ar parea ca nu are nici un sens sa spunem asa ceva. Sau poate nu inteleg bine.

Titlu: Măsurarea temperaturii atmosferice și punerea în evidență a schimbărilor climatice
Scris de: tavy din Ianuarie 31, 2011, 09:27:02 PM
Citat din: razvanc87 din Ianuarie 31, 2011, 09:04:09 PM
Bun, am inteles nemultumirile voastre legate de acest chestionar. Si in mare parte sunt de acord cu voi, dar nu sunt de acord cu idei precum Institutul National de C-D pentru Optoelectronica nu ar avea un departament care studiaza mediul, din moment ce eu fac parte din acel departament si de-asta am si postat chestionarul pe forumurile mai populare pe care le stiu.
Nu am spus că ,,Institutul National de C-D pentru Optoelectronica" nu ar avea un departament de mediu am spus doar că este dubios ca un institut de Optoelectronică să aibe un departamant de mediu.
Citat din: razvanc87 din Ianuarie 31, 2011, 09:04:09 PM
Ca sa vedeti comparatie intrati pe http://rado.inoe.ro/Quicklooks
Acesta este un site facut de mine, tot pentru RADO dupa cum se vede si in nume, care este pe un domeniu al INOE dupa cum se vede si din nume. Si sa zici ca un Institut National e de balta... asta denota dezinformare, din moment ce se afla pe cea mai mare platforma de institute din Romania si anume Magurele.
Din câte știu în Măgurele sunt și institute serioase: ISS, IFTM, IFIN.
Bănuiesc că ai teminat fizică, cum ai făcut în anul 3 ,,Termodinamică și fizică statistică" care este părerea ta referitoare la medierea mărimilor intensive?
Citat din: mircea_p din Ianuarie 31, 2011, 09:10:13 PM
Citat din: tavy din Ianuarie 31, 2011, 09:02:09 PM
Nu știu ce studii ai, poate nu ai cunoștințe de fizică dar temperatura este o mărime fizică intensivă, spre deosebire de mărimile fizice extensive media marimilor fizice intensive nu are nici o semnificație fizică. Cu alte cuvinte orice studiu care vorbește despre ,,temperatura medie" nu este un studiu stiințific. Oricum încălzirea nu este obligatoriu legată de creșterea temperaturii, în realitate un sistem se încălzește atunci când căldura primită este mai mare decât căldura cedata. Poți spre exemplu să încălzești (cvasistatic) un vas cu amestec de apă și ghiață la echilibru și temperatura acestuia nu se va modifica până nu se va topi toată ghiața.
Esti de acord cel putin ca vara e mai cald decat iarna?
Dupa ce spui aici s-ar parea ca nu are nici un sens sa spunem asa ceva. Sau poate nu inteleg bine.
Nu te oprește nimeni să compari temperaturile maxime sau minime din cele două perioade sau numărul de zile în care temperatura depășește un anumit prag și de aici să deduci dacă o perioadă este mai caldă decât alta. La limită, dar foarte la limită, poți opera cu media temperaturilor dar fără a numi acea medie temperatură, nici chiar temperatură medie, pentru că nu este o temperatură și oricum nu are nici o semnificație fizică.
Încă odată:
[tex]MediaTemperaturilor \neq TemperaturaMedie[/tex]
,,Temperatura medie" nu există.
Titlu: Măsurarea temperaturii atmosferice și punerea în evidență a schimbărilor climatice
Scris de: Adi din Ianuarie 31, 2011, 09:27:29 PM
Citat din: mircea_p din Ianuarie 31, 2011, 09:10:13 PM
Citat din: tavy din Ianuarie 31, 2011, 09:02:09 PM
Nu știu ce studii ai, poate nu ai cunoștințe de fizică dar temperatura este o mărime fizică intensivă, spre deosebire de mărimile fizice extensive media marimilor fizice intensive nu are nici o semnificație fizică. Cu alte cuvinte orice studiu care vorbește despre ,,temperatura medie" nu este un studiu stiințific. Oricum încălzirea nu este obligatoriu legată de creșterea temperaturii, în realitate un sistem se încălzește atunci când căldura primită este mai mare decât căldura cedata. Poți spre exemplu să încălzești (cvasistatic) un vas cu amestec de apă și ghiață la echilibru și temperatura acestuia nu se va modifica până nu se va topi toată ghiața.
Esti de acord cel putin ca vara e mai cald decat iarna?
Dupa ce spui aici s-ar parea ca nu are nici un sens sa spunem asa ceva. Sau poate nu inteleg bine.

Nici eu nu sunt de acord ca nu are sens sa vorbesti de temperatura medie. Tocmai, asta e cheia, sa calculezi temperatura medie atat in spatiu cat si in timp (desigur, nu medie aritmetica, ci una ponderata) pentru ca doar asa poti studia evolutia temperaturii medii globale si anuale.
Titlu: Măsurarea temperaturii atmosferice și punerea în evidență a schimbărilor climatice
Scris de: Adi din Ianuarie 31, 2011, 09:30:56 PM
Citat din: tavy din Ianuarie 31, 2011, 09:27:02 PM
Încă odată:
[tex]MediaTemperaturilor \neq TemperaturaMedie[/tex]
,,Temperatura medie" nu există.

OK, daca tii mortis, atunci nu vorbim de temperatura medie globala, ci de media in spatiu a temperaturilor inregistrate de multe statii meteorologice de pe planeta. Si da, nu este medie arithmetica, ci medie ponderata dupa anumite criterii.

Acum ca am lamurit cu terminologia, esti de acord ca evolutia in timp a mediei temperaturilor globale este marimea care ne indica daca temperatura se incalzeste sau se raceste?
Titlu: Măsurarea temperaturii atmosferice și punerea în evidență a schimbărilor climatice
Scris de: AlexandruLazar din Ianuarie 31, 2011, 09:34:31 PM
Mă gândeam şi eu că e logic că atunci când spun temperatură medie, mă refer la faptul că se adună ţ temperaturi şi se împarte la ţ, cu sau fără ponderare. Media temperaturilor pare să se măsoare tot în grade (de care-or fi), deci aş fi zis că-i tot temperatură. Bun, să îmi fie cu iertare gafa de semantică, vroiam să spun că media temperaturilor pare să fi crescut, ceea ce per ansamblu pentru mine, ca inginer metrolog de formaţie, înseamnă că-i mai cald. Sau greşesc şi e de fapt mai frig?
Titlu: Măsurarea temperaturii atmosferice și punerea în evidență a schimbărilor climatice
Scris de: Adi din Ianuarie 31, 2011, 09:35:54 PM
Razvan, apreciez comentariul tau detaliat, e foarte util. Si ma bucur ca faceti acele masuratori si le promovati si catre poporul roman, care nu stie ca la noi se mai face si cercetare, mai rar, dar se face.
Titlu: Măsurarea temperaturii atmosferice și punerea în evidență a schimbărilor climatice
Scris de: Adi din Ianuarie 31, 2011, 09:37:01 PM
Citat din: AlexandruLazar din Ianuarie 31, 2011, 09:34:31 PM
Mă gândeam şi eu că e logic că atunci când spun temperatură medie, mă refer la faptul că se adună ţ temperaturi şi se împarte la ţ, cu sau fără ponderare. Media temperaturilor pare să se măsoare tot în grade (de care-or fi), deci aş fi zis că-i tot temperatură. Bun, să îmi fie cu iertare gafa de semantică, vroiam să spun că media temperaturilor pare să fi crescut, ceea ce per ansamblu pentru mine, ca inginer metrolog de formaţie, înseamnă că-i mai cald. Sau greşesc şi e de fapt mai frig?

Excelent post! Complet de acord cu tine, ai formulat mai clar ca mine.
Titlu: Măsurarea temperaturii atmosferice și punerea în evidență a schimbărilor climatice
Scris de: tavy din Ianuarie 31, 2011, 09:40:33 PM
Citat din: Adi din Ianuarie 31, 2011, 09:27:29 PM
Nici eu nu sunt de acord ca nu are sens sa vorbesti de temperatura medie. Tocmai, asta e cheia, sa calculezi temperatura medie atat in spatiu cat si in timp (desigur, nu medie aritmetica, ci una ponderata) pentru ca doar asa poti studia evolutia temperaturii medii globale si anuale.
Adi, în mod sigur știi că temperatura este definită numai pentru sisteme la echilibru, atunci când măsurăm temperatura atmosferică forțăm puțin și presupunem că zona în care măsurăm este foarte aproape de echilibru, de aici și până a vorbi despre o temperatură globală este distanță mare pentru că atmosfera, la nivel global, este foarte departe de un sistem la echilibru.

Citat din: Adi din Ianuarie 31, 2011, 09:30:56 PM
OK, daca tii mortis, atunci nu vorbim de temperatura medie globala, ci de media in spatiu a temperaturilor inregistrate de multe statii meteorologice de pe planeta. Si da, nu este medie arithmetica, ci medie ponderata dupa anumite criterii.

Acum ca am lamurit cu terminologia, esti de acord ca evolutia in timp a mediei temperaturilor globale este marimea care ne indica daca temperatura se incalzeste sau se raceste?
După ce criterii am stabili respectivele ponderi? Dacă ponderile respective sunt capacități calorice atunci media respectivă nu mai are dimensiune de temperatură și chiar începe să aibă sens (deja este vorba de media unor energii, energia este extensivă și nu mai avem o problemă). Am motive serioase să consider că nu avem capacitatea să măsurăm dacă energia internă a atmosferei crește sau scade în cazul unor variații relativ mici cum sunt cele din, să zicem, ultima 100 de ani. Ar trebui să avem mult mai multe stații meteo decât în prezent pentru a face măsurători cu relevanță globală pentru un sistem atât de haotic cum este atmosfera.
Titlu: Măsurarea temperaturii atmosferice și punerea în evidență a schimbărilor climatice
Scris de: Adi din Ianuarie 31, 2011, 09:49:01 PM
Acum inteleg unde bati. Vrei sa intelegi mai bine metodologia lor de lucru, adica a celor de la NASA si institutul interguvernamental pentru schimbari climatice. Te invit intr-adevar sa le studiezi si sa vezi cum calculeaza ponderile. Din ce inteleg eu in mare este ca ponderea e proportionala cu volumul sau aria sau cantitatea de aer sau ceva in gen pentru care presupui ca au aceeasi temperatura care e data corect de temperatura masurata de statie. De exemplu, in SUA sunt mai multe statii decat in Africa pe suta de metri patrati.

Cred ca poti presupune ca sistemul e in echilibru pe distante mici.

Oricum, fara a fi expert in astea, am incredere in cei de la NASA, ca s-au gandit la absolut toate problemele ridicate de tine, ba inca 100 de probleme in plus, si le-au rezolvat pe toate.
Titlu: Măsurarea temperaturii atmosferice și punerea în evidență a schimbărilor climatice
Scris de: tavy din Ianuarie 31, 2011, 10:06:56 PM
Citat din: AlexandruLazar din Ianuarie 31, 2011, 09:34:31 PM
Mă gândeam şi eu că e logic că atunci când spun temperatură medie, mă refer la faptul că se adună ţ temperaturi şi se împarte la ţ, cu sau fără ponderare. Media temperaturilor pare să se măsoare tot în grade (de care-or fi), deci aş fi zis că-i tot temperatură. Bun, să îmi fie cu iertare gafa de semantică, vroiam să spun că media temperaturilor pare să fi crescut, ceea ce per ansamblu pentru mine, ca inginer metrolog de formaţie, înseamnă că-i mai cald. Sau greşesc şi e de fapt mai frig?
Într-o cameră ai 1 oră 25°C și o oră -25°C, în altă cameră ai în același timp 0°C. Poți să-mi spui făcând media temperaturilor în care cameră a fost mai cald? Și de ce. La fel, dacă consideri că în cele două camere a fost la fel de cald, de ce?
Dacă în două camere ai temperaturile -25°C respectiv 25°C poți spune care este media temperaturilor și care este relevanța acesteia?

Citat din: Adi din Ianuarie 31, 2011, 09:49:01 PM
Acum inteleg unde bati. Vrei sa intelegi mai bine metodologia lor de lucru, adica a celor de la NASA si institutul interguvernamental pentru schimbari climatice. Te invit intr-adevar sa le studiezi si sa vezi cum calculeaza ponderile. Din ce inteleg eu in mare este ca ponderea e proportionala cu volumul sau aria sau cantitatea de aer sau ceva in gen pentru care presupui ca au aceeasi temperatura care e data corect de temperatura masurata de statie. De exemplu, in SUA sunt mai multe statii decat in Africa pe suta de metri patrati.
Sunt convins că densitatea stațiilor meteo din SUA este mult prea mică pentru acest scop. Chiar dacă ar fi suficientă tot nu ar fi relevant raportat la suprafața terei.

Citat din: Adi din Ianuarie 31, 2011, 09:49:01 PM
Cred ca poti presupune ca sistemul e in echilibru pe distante mici.
Ce înțelegi prin distanțe mici, 1km este deja foarte mult și pentru asta este suficient să privești vara pe fereastră de la un etaj ceva mai mare când vine o furtună.

Citat din: Adi din Ianuarie 31, 2011, 09:49:01 PM
Oricum, fara a fi expert in astea, am incredere in cei de la NASA, ca s-au gandit la absolut toate problemele ridicate de tine, ba inca 100 de probleme in plus, si le-au rezolvat pe toate.
Nu trebuie să fii expert, aplica doar ce ai învățat la termodinamică și o să vezi că ceva scârțâie. La urma urmei și NASA are nevoie de fonduri și din studiile cu privire la ,,încălzirea globală" se obțin destul de ușor aceste fonduri.
Mai gândeștete și la faptul că toate modelele care prognozau răcire/încălzire catastrofică au fost infirmate de observațiile ulterioare. Acum 30 de ani se prognoza o răcire globală, găsiseră și vinovatul, pulberile emanate de industrie care reduce energia solară care atinge solul, până de curând se prognoza încălzire globală, vinovatul fiind dioxidul de carbon, în ultima vreme se vorbește tot mai rar de încălzire globală și se vorbește tot mai mult despre schimbări climatice. Ba chiar am văzut pe la televizor indivizi care nu au aflat de schimbarea de paradigmă, ,,incălzire globală" vs. ,,schimbări climatice" și  încercau să mă convingă că gerul din ultima vreme este din cauza încălzirii globale.

P.S.
Dacă deranjez sper să fiu avertizat înainte să fiu banat, promit că mă opresc.
Știu că am tendința să fiu mai acid, nu o fac din răutate, așa sunt eu nu vă supărați pe mine.
Titlu: Măsurarea temperaturii atmosferice și punerea în evidență a schimbărilor climatice
Scris de: Adi din Ianuarie 31, 2011, 10:13:47 PM
Comentariile tale sunt binevenite si acum ai adus critici inca mai constructive ca inainte. Nu vei fi banat. Suntem aici sa discutam stiinta, iar stiinta se face prin dialog si polemica constructiva.

La prima ta intrebare as raspunde ca media temperaturilor a fost zero in prima camera si zero in cea de-a doua. Camera in medie nici nu s-a racit, nici nu s-a marit. Relevanta e cu anotimpurile. Ca atunci cand cand e iarna intr-o emisfera e vara in alta emisfera si ca se poate ca iarna sa fie mai rece in nord si mai calda in sud si totusi media temperaturilor globala sa fi crescut, adica sa fie mai multa energie totala in atmosfera, care apoi va provoca mai multe urgane si da, schimbari climatice. Asta pentru ca atmosfera a absorbit mai multa energie de la Soare datorita efectului de seara datorita CO2 din atmosfera. Nu inteleg care parte din asta nu e clar. Pana la urma e irelavant daca media temperaturilor creste sau nu, cat conteaza energia disponibila in atmosfera si in oceane. Si acesta energie creste incontinuu, iar dupa ce se va topi gheta de la poli, energia se va duce direct in cresterea de temperatura in mod mai accelerat.
Titlu: Măsurarea temperaturii atmosferice și punerea în evidență a schimbărilor climatice
Scris de: razvanc87 din Ianuarie 31, 2011, 10:27:39 PM
tavy, sunt de acord cu tine, modul tau de privire a lucrurilor care este foarte sceptic este bine fondat. Motivul pentru care "incalzirea globala" si "schimbarea climatica" atrage foarte multe fonduri este datorat faptului ca atmosfera este un sistem atat de complex care este de asemenea pus in contact cu hidrosfera si cu litosfera si cu tot ce mai misca acolo. E foarte greu sa faci progrese in 5 ani... si 50 de ani e mult (Edit: scuze, vroiam sa zic putin), si la fel ca in alte domenii, mergi pe speculatii, mai intai vine parerea si apoi vine confirmarea. Dar asta inseamna progres, cel putin din punctul meu de vedere... si pana la urma ce conteaza daca aceste tipuri de investigatii absorb bani? Chiar crezi ca este degeaba?
Titlu: Măsurarea temperaturii atmosferice și punerea în evidență a schimbărilor climatice
Scris de: tavy din Ianuarie 31, 2011, 10:41:05 PM
Citat din: Adi din Ianuarie 31, 2011, 10:13:47 PM
Comentariile tale sunt binevenite si acum ai adus critici inca mai constructive ca inainte. Nu vei fi banat. Suntem aici sa discutam stiinta, iar stiinta se face prin dialog si polemica constructiva.

La prima ta intrebare as raspunde ca media temperaturilor a fost zero in prima camera si zero in cea de-a doua. Camera in medie nici nu s-a racit, nici nu s-a marit. Relevanta e cu anotimpurile. Ca atunci cand cand e iarna intr-o emisfera e vara in alta emisfera si ca se poate ca iarna sa fie mai rece in nord si mai calda in sud si totusi media temperaturilor globala sa fi crescut, adica sa fie mai multa energie totala in atmosfera, care apoi va provoca mai multe urgane si da, schimbari climatice. Asta pentru ca atmosfera a absorbit mai multa energie de la Soare datorita efectului de seara datorita CO2 din atmosfera. Nu inteleg care parte din asta nu e clar. Pana la urma e irelavant daca media temperaturilor creste sau nu, cat conteaza energia disponibila in atmosfera si in oceane. Si acesta energie creste incontinuu, iar dupa ce se va topi gheta de la poli, energia se va duce direct in cresterea de temperatura in mod mai accelerat.
Poți să-mi dai un exemplu despre cum ar fi legată media temperaturilor globală de energia totală în atmosferă? Eu nu pot să-mi imaginez o astfel de legătură în condițiile în care pot imagina sisteme a căror temperatură să crească scăzândule energia internă și sisteme a căror temperatură să crească în timp ce energia internă scade, în transformări cvasistatice și pentru sisteme apropiate de echilibrul termodinamic pentru care pot să vorbesc de temperatură. Nu numai că media temperaturilor mă pune în încurcătură, dar nici măcar nu-mi amintesc vre-o singură formulă în toată fizica învățată de mine în care să am de-a face cu adunări/scăderi de temperaturi nici chiar înmulțite cu un factor adimensional (nu iau aici în calcul formulele în care se da factor comun capacitatea calorică pentru că acolo avem de-a face de fapt cu adunări/scăderi de energii iar factorul comun nu este decât un artificiu). Ți-aș fi recunoscător dacă mi-ai da un exemplu de astfel de formulă ținând cont că eu nu am mai avut de-a face cu fizica de 15 ani.

Citat din: razvanc87 din Ianuarie 31, 2011, 10:27:39 PM
tavy, sunt de acord cu tine, modul tau de privire a lucrurilor care este foarte sceptic este bine fondat. Motivul pentru care "incalzirea globala" si "schimbarea climatica" atrage foarte multe fonduri este datorat faptului ca atmosfera este un sistem atat de complex care este de asemenea pus in contact cu hidrosfera si cu litosfera si cu tot ce mai misca acolo. E foarte greu sa faci progrese in 5 ani... si 50 de ani e mult, si la fel ca in alte domenii, mergi pe speculatii, mai intai vine parerea si apoi vine confirmarea. Dar asta inseamna progres, cel putin din punctul meu de vedere... si pana la urma ce conteaza daca aceste tipuri de investigatii absorb bani? Chiar crezi ca este degeaba?
Aș fi și eu de acord cu tine dacă nu ai încerca să modelezi un sistem haotic, numeric bineînțeles că analitic este evident că nu se poate, folosind ca date de intrare masură un grid mult prea mare. Dacă ai avea la dispoziție măsurători cu un grid de, să zicem, 10m, pe toată suprafața pământului, și o putere de calcul de câteva milioane de ori puterea de calcul a tuturor calculatoarelor din acest moment de pe pământ, atunci te-aș crede că ai șanse să folosești cu succes acele date. În momentul acesta gridul este mult mai mare decât cei 10m iar în anumite zone are peste 1000km. La acest grid nici nu ai nevoie de puterea de calcul la care mă refeream eu dar nici șanse să poți face predicții credibile, poți totuși ca, din când în când, să o nimerești.
Mai mult, cum aș putea avea încredere în modelele climatologilor când am văzut astfel de modele în care se lua în calcul concentrația dioxidului de carbon în atmosferă dar nu se lua în calcul variația activității soarelui, umiditatea atmosferică sau solubilitatea dioxidului de carbon în apa oceanelor.
Titlu: Măsurarea temperaturii atmosferice și punerea în evidență a schimbărilor climatice
Scris de: Adi din Ianuarie 31, 2011, 10:47:38 PM
Deocamdata pot spune ca sunt de acord ca atunci cand va fi un grid mult mai dens si masuratoarea va fi mai precisa. Inca nu sunt convins ca acum nu e suficient de precisa, adica nu sunt convins ca roarea de masurare e mai mare decat cei 0.3 grade Celsius cu cat se zice ca a crescut temperatura de dinainte de revolutia industriala pana acum.
Titlu: Măsurarea temperaturii atmosferice și punerea în evidență a schimbărilor climatice
Scris de: razvanc87 din Ianuarie 31, 2011, 10:59:06 PM
Ai fi surprins cat de multe lucruri se pot intelege prin simplificari. Ceea ce spui tu cu supercalculatoare, mai mult ca sigur ca ai dreptate, nu ai cum sa prezici asemenea lucruri avand in vedere stadiul actual de dezvoltare. Dar, si e un mare DAR, poti intelege calitativ anumite aspecte. Nu cred ca e nevoie de o grila de 10 pe 10 metri ca sa iti dai seama ca ghetarii intr-adevar se topesc. Acum ca asta o fi din cauza "incalzirii globale", "schimbarii climatice", oamenilor, ornitorincilor ori oricui altcineva, nu se cunoaste, dar se poate presupune prin simplificari (care dau gres intr-un sistem asa complex de cele mai multe ori). Dar din cand in cand se mai gaseste cate un experiment care iti confirma ceea ce vrei sa stii.

Mai nou exista sateliti care iti pot da informatii si despre temperatura... si da, exista multe erori in acele masuratori, dar sunt probabil cele mai precise masuratori pe care le avem in momentul actual la scara globala (ele fiind pe grile si de cateva sute de metri pe sute de metri), dar iti dai seama ca pentru moment nu se gandeste nimeni sa foloseasaca o asemenea grila din moment ce ar dura probabil o viata numai sa se faca acele calcule de catre calculatoare actuale.

Ca exemplu de sistem foarte complex, pentru care s-au facut metode sofisticate de calcule si au esuat este sistemul oceanelor si mai precis circulatia termohalina. Dar, cu toate ca acele calcule complexe au esuat, un model a iesit in evidenta prin simplitatea ei si faptul ca intr-un mod calitativ a reusit sa reproduca ciclul acestei circulatii, dovedit experimental (mai precis cu ajutorul datelor paleoclimatice). (Stommel a fost cel care a propus acest model http://www.pik-potsdam.de/~stefan/thc_fact_sheet.html - acest link este gasit pe google, nu am o sursa mai buna, dar pare a avea ca referinta titlurile relevante).
Titlu: Măsurarea temperaturii atmosferice și punerea în evidență a schimbărilor climatice
Scris de: AlexandruLazar din Ianuarie 31, 2011, 11:10:27 PM
Citat din: tavy din Ianuarie 31, 2011, 10:06:56 PM
Citat din: AlexandruLazar din Ianuarie 31, 2011, 09:34:31 PM
Mă gândeam şi eu că e logic că atunci când spun temperatură medie, mă refer la faptul că se adună ţ temperaturi şi se împarte la ţ, cu sau fără ponderare. Media temperaturilor pare să se măsoare tot în grade (de care-or fi), deci aş fi zis că-i tot temperatură. Bun, să îmi fie cu iertare gafa de semantică, vroiam să spun că media temperaturilor pare să fi crescut, ceea ce per ansamblu pentru mine, ca inginer metrolog de formaţie, înseamnă că-i mai cald. Sau greşesc şi e de fapt mai frig?
Într-o cameră ai 1 oră 25°C și o oră -25°C, în altă cameră ai în același timp 0°C. Poți să-mi spui făcând media temperaturilor în care cameră a fost mai cald? Și de ce. La fel, dacă consideri că în cele două camere a fost la fel de cald, de ce?
Dacă în două camere ai temperaturile -25°C respectiv 25°C poți spune care este media temperaturilor și care este relevanța acesteia?

Nu discutăm despre media temperaturilor a *două* camere la un moment dat ci de felul cum evoluează media asta în timp pentru un număr foarte mare de camere. Dacă ai o clădire cu 500 de camere si observi că prin medierea temperaturilor, în condiţii similare de iluminare solară, conductivitate termică a zidurilor ş.a.m.d., de la o zi la alta, valoarea obţinută e tot mai mare, mie mi se pare o presupunere rezonabilă să spui că în majoritatea camerelor se face mai cald, nu că în 490 temperatura rămâne la fel şi în celelalte 10 temperatura creşte cu 200 de grade ca să dea media mai mare. Chiar dacă asta s-ar putea întâmpla într-o clădire, mi-e mai greu să cred cum s-ar face asta într-o atmosferă.

Sunt aceleaşi metode de investigaţie care se folosesc şi pentru studiul zonelor de climă de exemplu. Media temperaturilor din ianuarie în mijlocul Siberiei e mai scăzută decât media temperaturilor din ianuarie din Kyoto, şi asta nu dintr-un capriciu al statisticii ci pur şi simplu pentru că, în majoritatea momentelor în care faci măsurătoarea, în mijlocul Siberiei e mai frig decât în Kyoto. La fel, dacă de la un an la altul, pentru un număr mare de puncte de măsurare şi de momente de măsurare, îţi iese că media temperaturilor de acum e mai mare decât acum 10 ani, eu cred că e rezonabil să presupui că asta e din cauză că în majoritatea cazurilor când ai făcut măsurătoarea, a fost mai cald. De-asta se şi fac mai multe măsurători şi de-asta se fac şi medieri.


Edit: ca să ne înţelegem, nu vreau să bat toba activismului şi să insinuez că trebuie să ne apucăm să trăim ca-n epoca de piatră, n-am nicio treabă cu oricare din ideile astea. Din partea mea poate să fie şi a ornitorincilor, însă cunoscând felul cum se fac măsurătorile de acest tip, mie mi se pare că o curbă ascendentă a mediilor de temperatură pe toate palierele de timp (lunar, sezonier, anual, bianual, 5 ani) în condiţiile în care dispersia hărţilor izoterme nu se modifică radical (adică nu sunt "camere" unde e bocnă şi "camere" unde te fierbi) sunt indicii rezonabile că e mai cald ca înainte.
Titlu: Măsurarea temperaturii atmosferice și punerea în evidență a schimbărilor climatice
Scris de: tavy din Ianuarie 31, 2011, 11:33:52 PM
Citat din: Adi din Ianuarie 31, 2011, 10:47:38 PM
Deocamdata pot spune ca sunt de acord ca atunci cand va fi un grid mult mai dens si masuratoarea va fi mai precisa. Inca nu sunt convins ca acum nu e suficient de precisa, adica nu sunt convins ca roarea de masurare e mai mare decat cei 0.3 grade Celsius cu cat se zice ca a crescut temperatura de dinainte de revolutia industriala pana acum.
Poate chiar cele 0,3°C ar trebui să te pună pe gânduri, cred că știi și tu că temperatura este marimea fizică fundamentală care o măsurăm cu cea mai mică precizie, măsurările meteo cred că nu se fac cu o precizie mai mare de 0,3°C, nu am date așa că nu sunt sigur de asta. Sunt însă convins că înainte de revoluția îndustrială eroarea de măsură era ceva mai mare. Chiar dacă am face măsurători mai precise, fluctuațiile termodinamice fac irelevante aceste precizii. În consecință cele 0,3°C se pot încadra lejer în limita de eroare a măsurătorii și devin nerelevanți.
Gândeștete că stațiile meteo se găsesc, în general, la distanțe de peste 10km, nu este neobișnuit ca la distanțe de mai puțin de 1km să ai diferențe de temperatură de peste 1°C.

Citat din: razvanc87 din Ianuarie 31, 2011, 10:59:06 PM
Ai fi surprins cat de multe lucruri se pot intelege prin simplificari. Ceea ce spui tu cu supercalculatoare, mai mult ca sigur ca ai dreptate, nu ai cum sa prezici asemenea lucruri avand in vedere stadiul actual de dezvoltare. Dar, si e un mare DAR, poti intelege calitativ anumite aspecte. Nu cred ca e nevoie de o grila de 10 pe 10 metri ca sa iti dai seama ca ghetarii intr-adevar se topesc. Acum ca asta o fi din cauza "incalzirii globale", "schimbarii climatice", oamenilor, ornitorincilor ori oricui altcineva, nu se cunoaste, dar se poate presupune prin simplificari (care dau gres intr-un sistem asa complex de cele mai multe ori). Dar din cand in cand se mai gaseste cate un experiment care iti confirma ceea ce vrei sa stii.
Mai ales când faci selecție observațională și nu ții cont de ghețarii care cresc.

Citat din: razvanc87 din Ianuarie 31, 2011, 10:59:06 PM
Mai nou exista sateliti care iti pot da informatii si despre temperatura... si da, exista multe erori in acele masuratori, dar sunt probabil cele mai precise masuratori pe care le avem in momentul actual la scara globala (ele fiind pe grile si de cateva sute de metri pe sute de metri), dar iti dai seama ca pentru moment nu se gandeste nimeni sa foloseasaca o asemenea grila din moment ce ar dura probabil o viata numai sa se faca acele calcule de catre calculatoare actuale.
Cu alte cuvinte nu avem puterea să prelucrăm datele de la un grid de sute de ori mai puțin dens decât necesar dar avem pretenția să obținem modele relevante de la un grid cu densități infinit mai mici.

Citat din: razvanc87 din Ianuarie 31, 2011, 10:59:06 PM
Ca exemplu de sistem foarte complex, pentru care s-au facut metode sofisticate de calcule si au esuat este sistemul oceanelor si mai precis circulatia termohalina. Dar, cu toate ca acele calcule complexe au esuat, un model a iesit in evidenta prin simplitatea ei si faptul ca intr-un mod calitativ a reusit sa reproduca ciclul acestei circulatii, dovedit experimental (mai precis cu ajutorul datelor paleoclimatice). (Stommel a fost cel care a propus acest model http://www.pik-potsdam.de/~stefan/thc_fact_sheet.html - acest link este gasit pe google, nu am o sursa mai buna, dar pare a avea ca referinta titlurile relevante).
Să înțeleg că modelul a eșuat cu succes în a face predictii dar a putut fi aranjat de așa natură încât să dea cu datele deja existente. Nu m-ar mira, la câte modele sunt, ca unul să nimerească și să facă predictii chiar pentru o perioadă destul de mare, problema e că sunt atât de multe modele încât nu ai cum să știi pe care să-l alegi. Este ca și cum ai arunca cu zarul și ai avea 6 modele de predicție care fiecare duce la un număr diferit, în mod evident unul din ele se va confirma dar asta nu înseamnă că este un model corect. Dacă am suficient de multe modele care produc numere aleatoare intre 1 și 6 atunci pot să arunc de foarte multe ori cu zarul și tot voi avea un model care să ,,ghicească" ce urmează.
La fel și cu modelele climatologice, sunt modele care prezic creșteri de temperatură cu 1°, 3°, 7° , 10°... (numere puse din burtă) altele scaderi de temperatură cu 1°, 3°, 7° , 10°... în următorii 10, 100, 1000 de ani. Este destul de puțin probabil ca nici una din predicții să nu se confirme pentru că este improbabil ca clima să nu se schimbe (oare or fi modele care să prezică că clima va rămâne neschimbată pentru prima oară în istorie că eu abia atunci mi-as pune ceva probleme).
Poți să spui că este la fel cu prezicerea cutremurelor, tot timpul câte cineva prezice un cutremur, chiar dacă în ultima vreme presa a înțeles că trebuie să-i ignore, până la urmă cineva va ghici un cutremur și va zice ,,v-am spus eu că în data de va fi cutremur!".
Din păcate știința nu merge așa, nu am văzut în știință, în altă parte decât în climatologie, să fie scoase teorii la normă și pe urmă să se vadă care se nimerește. În general teoriile științifice apar rar, au o logică bine stabilită și sunt bine verificate.

Citat din: AlexandruLazar din Ianuarie 31, 2011, 11:10:27 PM
Nu discutăm despre media temperaturilor a *două* camere la un moment dat ci de felul cum evoluează media asta în timp pentru un număr foarte mare de camere. Dacă ai o clădire cu 500 de camere si observi că prin medierea temperaturilor, în condiţii similare de iluminare solară, conductivitate termică a zidurilor ş.a.m.d., de la o zi la alta, valoarea obţinută e tot mai mare, mie mi se pare o presupunere rezonabilă să spui că în majoritatea camerelor se face mai cald, nu că în 490 temperatura rămâne la fel şi în celelalte 10 temperatura creşte cu 200 de grade ca să dea media mai mare. Chiar dacă asta s-ar putea întâmpla într-o clădire, mi-e mai greu să cred cum s-ar face asta într-o atmosferă.
Realizezi că modelul cu cele 500 de camere este la fel de depărtat de modelul atmosferic ca și modelul meu cu două camere. Atmosfera nu este nici izocoră, radiația solară variază în limite largi în cadrul ciclurilor solare ba chiar și pe perioade mai mari, echivalentul conductivității zidurilor este dat atât de curenții de aer cât și de cei oceanici, atmosfera este foarte departe de echilibru termodinamic iar diferențele de temperatură între zonele geografice, între zi și noapte și între anotimpuri sunt destul de mari, nu departe de diferențele de temperatură dintre cele două camere din exemplul meu.
Exemplul meu cu cele două camere nu încerca să fie un model pentru admosferă ci doar să arate cât de irelevantă este media temperaturilor, oricum temperaturile se adună (pentru a da media) doar în climatologie, din câte știu eu, dar poate îmi dai tu un exemplu din fizică unde apare într-o formulă suma, eventual cu ceva ponderi, a unor temperaturi.
Titlu: Măsurarea temperaturii atmosferice și punerea în evidență a schimbărilor climatice
Scris de: razvanc87 din Ianuarie 31, 2011, 11:51:24 PM
Cu alte cuvinte vrei sa spui ca mai intai se dezvolta teoria dupa care se aplica? sau ca asa ar trebui sa fie lucrurile? Daca asa ar fi fost, atunci progresul tehnologic ar fi tins catre 0. Cel putin din cate mi-au fost date exemple, mai intai se propunea a se face o incercare (sa zicem in toria starii solide, crearea unei celule fotovoltaice - se combina tot felul de materiale, Cadmiu, Telur, Aluminiu etc.) si apoi se caracterizeaza si se "produce" teoria. Foarte rar se intampla invers, adica sa cercetezi teoria si apoi sa creezi ceva sau sa analizezi ceva pe baza ei.
Titlu: Măsurarea temperaturii atmosferice și punerea în evidență a schimbărilor climatice
Scris de: mircea_p din Februarie 01, 2011, 12:01:53 AM
Citat din: tavy din Ianuarie 31, 2011, 09:27:02 PM
Încă odată:
[tex]MediaTemperaturilor \neq TemperaturaMedie[/tex]
,,Temperatura medie" nu există.
Am zis eu (sau altcineva aici) ca ar fi adevarata egalitatea asta? Nu inteleg de ce trebuie sa o scrii asa cu litere mari. ???

De acord ca temperatura medie nu este o temperatura in sens termodinamic.
Dar asta nu inseamna ca nu are sens. La fel cum media unei clase se poate intampla sa nu fie egala cu nota nici unui copil din clasa. Deci nu eate o nota in sensul propriu. Totusi este o indicatie relevanta asupra nivelului clasei si se poate folosi pentru a face comparatii.

Dar vad care este problema pe care o pui mai ales ca si eu mi-am pus intrebeari asemanatoare in privinta felului cum se calculeaza marimea numita (daca este numita asa) temperatura medie.

Nota
(Sorry, am pus din greseala semnul de "angry" in loc de mirat. Scuze daca a vazut cineva asta, inainte sa il schimb.Nu voiam nicidecum sa pun asa ceva; doar  nedumerit)
Titlu: Măsurarea temperaturii atmosferice și punerea în evidență a schimbărilor climatice
Scris de: Adi din Februarie 01, 2011, 12:04:14 AM
Citat din: mircea_p din Februarie 01, 2011, 12:01:53 AM
La fel cum media unei clase se poate intampla sa nu fie egala cu nota nici unui copil din clasa. Deci nu eate o nota in sensul propriu. Totusi este o indicatie relevanta asupra nivelului clasei si se poate folosi pentru a face comparatii.

Excelent, exemplu, nu ma gandisem la el.
Titlu: Măsurarea temperaturii atmosferice și punerea în evidență a schimbărilor climatice
Scris de: AlexandruLazar din Februarie 01, 2011, 12:08:49 AM
CitatPoate chiar cele 0,3°C ar trebui să te pună pe gânduri, cred că știi și tu că temperatura este marimea fizică fundamentală care o măsurăm cu cea mai mică precizie, măsurările meteo cred că nu se fac cu o precizie mai mare de 0,3°C, nu am date așa că nu sunt sigur de asta.

Nici vorbă. Măsurători de calitate industrială (d-aia de fabrici şi uzine) se pot face cu precizie de 0.3°C de cel puţin 20 de ani. Măsurarea în staţii meteo se poate face fără probleme la precizii de 0.1°C, iar cele din staţiile de studiu climatologice se fac cu incertitudini până spre 0.03-0.01°C cu atât mai mult cu cât nu se face o singură măsurare şi nu se face în orice condiţii. Se folosesc mai multe termometre (de regulă cel puţin 8 ), plasate în incinte speciale. Edit: ca idee, măsurarea cu incercitudini de 0.1°C a temperaturii era o chestie care se putea face rezonabil de uşor la nivelul anilor '50-'60. Un senzor RTD cu incertitudine de de 0.1°C costă la ora actuală câţiva euro. Desigur, precizia lanţului de măsurare de după el trebuie să fie foarte ridicată (ăsta e de fapt mai scump  ;D) dar asta nu e o problemă atât de mare în măsurătorile climatice, unde domeniul de măsurare nu e foarte mare.

CitatExemplul meu cu cele două camere nu încerca să fie un model pentru admosferă ci doar să arate cât de irelevantă este media temperaturilor, oricum temperaturile se adună (pentru a da media) doar în climatologie, din câte știu eu, dar poate îmi dai tu un exemplu din fizică unde apare într-o formulă suma, eventual cu ceva ponderi, a unor temperaturi.

Păi asta încerc şi eu să spun. Nu discutăm de cât de relevantă e medierea a trei temperaturi în decursul a două zile într-o incintă sferică perfect izolată plasată în vid. Discutăm de medierea unui număr foarte mare de temperaturi (ceea ce netezeşte vârfurile), pe un număr mare de paliere temporare, în zone extrem de diferite şi toate ies mai sus, indiferent că e vorba de media temperaturilor din Sahara sau de cea de la Polul Sud. Nu mai vorbim de coroborarea cu datele celelalte, cum ar fi de exemplu rata de topire a tuturor gheţarilor (ceea ce exclude probleme legate de compoziţia chimică, structura geologică sau presiunea atmosferică), care cred că arată oricum în mod destul de convingător că per ansamblu într-un an numărul zilelor destul de calde ca să se topească gheaţa e mai mare decât era înainte.

Edit: eh, apropo de asta:

CitatDar vad care este problema pe care o pui mai ales ca si eu mi-am pus intrebeari asemanatoare in privinta felului cum se calculeaza marimea numita (daca este numita asa) temperatura medie.

Da, chiar i se zice aşa prin orice context legat de climatologie sau meteorologie. Serios acuma, spune cineva întotdeauna  "intensitate a curentului electric" în loc de "curent electric" sau şi mai rău, "amperaj"?
Titlu: Măsurarea temperaturii atmosferice și punerea în evidență a schimbărilor climatice
Scris de: mircea_p din Februarie 01, 2011, 12:21:25 AM
Citat din: AlexandruLazar din Februarie 01, 2011, 12:08:49 AM

Edit: eh, apropo de asta:

CitatDar vad care este problema pe care o pui mai ales ca si eu mi-am pus intrebeari asemanatoare in privinta felului cum se calculeaza marimea numita (daca este numita asa) temperatura medie.

Da, chiar i se zice aşa prin orice context legat de climatologie sau meteorologie. Serios acuma, spune cineva întotdeauna  "intensitate a curentului electric" în loc de "curent electric" sau şi mai rău, "amperaj"?
Nu inteleg cum e "apropo" la ce spuneai mai sus.
Eu nu am verificat daca se numeste (sau foloseste) temperatura medie, la asta se referea paranteza.

Dar oricum ar numi-o, nu are importanta atat timp cat e clar despre ce e vorba.


Titlu: Măsurarea temperaturii atmosferice și punerea în evidență a schimbărilor climatice
Scris de: tavy din Februarie 01, 2011, 12:25:57 AM
Citat din: razvanc87 din Ianuarie 31, 2011, 11:51:24 PM
Cu alte cuvinte vrei sa spui ca mai intai se dezvolta teoria dupa care se aplica? sau ca asa ar trebui sa fie lucrurile? Daca asa ar fi fost, atunci progresul tehnologic ar fi tins catre 0. Cel putin din cate mi-au fost date exemple, mai intai se propunea a se face o incercare (sa zicem in toria starii solide, crearea unei celule fotovoltaice - se combina tot felul de materiale, Cadmiu, Telur, Aluminiu etc.) si apoi se caracterizeaza si se "produce" teoria. Foarte rar se intampla invers, adica sa cercetezi teoria si apoi sa creezi ceva sau sa analizezi ceva pe baza ei.
Vreau să spun că se adună date, se vine cu o teorie (sau foarte puține teorii) care corespunde cu datele și se verifică dacă teoria face predictii valide, dacă teoria nu face predictii valide este îmbunătățită sau abandonată în favoarea alteia, ciclul se reia.
În climatologie: se adună date, se emit 100.000 de teorii care corespund cu datele adunate, se selectează din cele 100.000 de teorii cele 1.000 de teorii care au facut predicții valide până la un moment dat și de mai vine cu încă 100.000 de teorii ca să nu se intre în criză de teorii în cazul în care la un moment dat și cele 1.000 din lotul anterior eșuează. Am exagerat puțin numerele dar nu prea mult. Din păcate în cazul teoriilor, selecția naturală nu funcționează.

Citat din: Adi din Februarie 01, 2011, 12:04:14 AM
Citat din: mircea_p din Februarie 01, 2011, 12:01:53 AM
La fel cum media unei clase se poate intampla sa nu fie egala cu nota nici unui copil din clasa. Deci nu eate o nota in sensul propriu. Totusi este o indicatie relevanta asupra nivelului clasei si se poate folosi pentru a face comparatii.

Excelent, exemplu, nu ma gandisem la el.
Întradevăr, exemplul este excelent, de mult timp căutam un exemplu de mărime intensivă care să nu fie la fel de abstractă ca temperatura. Într-adevăr media notelor este tot atât de relevantă ca media temperaturilor. Nota 2 plus nota 3 face nota 5 la fel cum 2°C plus 3°C fac 5°C. Si o clasă cu elevi cu IQ 100 este echivalentă cu o clasă în care o treime din elevi au IQ 140 iar două treimi au IQ 80.

Citat din: AlexandruLazar din Februarie 01, 2011, 12:08:49 AM
CitatPoate chiar cele 0,3°C ar trebui să te pună pe gânduri, cred că știi și tu că temperatura este marimea fizică fundamentală care o măsurăm cu cea mai mică precizie, măsurările meteo cred că nu se fac cu o precizie mai mare de 0,3°C, nu am date așa că nu sunt sigur de asta.

Nici vorbă. Măsurători de calitate industrială (d-aia de fabrici şi uzine) se pot face cu precizie de 0.3°C de cel puţin 20 de ani. Măsurarea în staţii meteo se poate face fără probleme la precizii de 0.1°C, iar cele din staţiile de studiu climatologice se fac cu incertitudini până spre 0.03-0.01°C cu atât mai mult cu cât nu se face o singură măsurare şi nu se face în orice condiţii. Se folosesc mai multe termometre (de regulă cel puţin 8 ), plasate în incinte speciale. Edit: ca idee, măsurarea cu incercitudini de 0.1°C a temperaturii era o chestie care se putea face rezonabil de uşor la nivelul anilor '50-'60. Un senzor RTD cu incertitudine de de 0.1°C costă la ora actuală câţiva euro. Desigur, precizia lanţului de măsurare de după el trebuie să fie foarte ridicată (ăsta e de fapt mai scump  ;D) dar asta nu e o problemă atât de mare în măsurătorile climatice, unde domeniul de măsurare nu e foarte mare.
Dacă ai citi un curs de fizică statistică ai afla despre fluctuațiile termodinamice și ai înțelege de ce măsurarea temperaturii în stațiile meteo cu erori de peste 0,5°C  este irelevantă. Dacă ai avea ceva noțiuni de calculul erorilor, se fac în general în anul întâi de facultate, ai ști că utilizarea mai multor termometre nu te ajută cu foarte mult. Nu degeaba temperatura este marimea fizică fundamentală care o măsurăm cu cea mai mică precizie, indiferent de condiții iar condițiile meteo sunt departe de condițiile ideale. Că să-ți faci o idee, dacă măsori temperatura cu miime de grad este o performanță char pentru un institut metrologic și asta se face în condiții bine determinate, puncte triple, chestii d-astea, pe când timplul îl măsurăm în mod curent cu precizii mult mai bune de 1020s.
Titlu: Măsurarea temperaturii atmosferice și punerea în evidență a schimbărilor climatice
Scris de: mircea_p din Februarie 01, 2011, 12:37:42 AM
Citat din: tavy din Februarie 01, 2011, 12:25:57 AM
Întradevăr, exemplul este excelent, de mult timp căutam un exemplu de mărime intensivă care să nu fie la fel de abstractă ca temperatura. Într-adevăr media notelor este tot atât de relevantă ca media temperaturilor. Nota 2 plus nota 3 face nota 5 la fel cum 2°C plus 3°C fac 5°C. Si o clasă cu elevi cu IQ 100 este echivalentă cu o clasă în care o treime din elevi au IQ 140 iar două treimi au IQ 80.
Nu stiu daca ai intentia sa fii sarcastic sau nu.
Dar ai spus-o foarte bine. Media clasei are o relevanta (sau irelevanta) similara cu media temperaturilor.
La fel cum intr-o clasa cu jumate tampiti si jumate genii poti avea o medie asa si asa, o temperatura medie  (in sensul discutat pana acum) temperata poate insemna niste extreme deosebite de tempratura.

Atata timp cat stim ce este media respectiva nu vad nici un pericol in a o folosi. Fie ca o numim temperatura medie sau medie a temperaturilor (stiu, nu sant egale).
In privinta relevantei pentru incalzirea globala, e alta poveste. Nu am studiat destul problema ca sa ma atac.
Titlu: Măsurarea temperaturii atmosferice și punerea în evidență a schimbărilor climatice
Scris de: AlexandruLazar din Februarie 01, 2011, 12:46:18 AM
CitatDacă ai avea ceva noțiuni de calculul erorilor, se fac în general în anul întâi de facultate, ai ști că utilizarea mai multor termometre nu te ajută cu foarte mult. Nu degeaba temperatura este marimea fizică fundamentală care o măsurăm cu cea mai mică precizie, indiferent de condiții iar condițiile meteo sunt departe de condițiile ideale. Că să-ți faci o idee, dacă măsori temperatura cu miime de grad este o performanță char pentru un institut metrologic și asta se face în condiții bine determinate, puncte triple, chestii d-astea

Sorry, acum îi vinzi castraveţi grădinarului ;D. Un senzor integrat, care vine în capsulă cu tot cu circuitul de condiţionare a semnalului, filtrare şi toate celelalte poate lucra comod la precizii de 0.1% în condiţii departe de cele ideale. N-o zic din aer ci din foaia de catalog: www.analog.com/static/imported-files/data_sheets/3B34.pdf . Cazul cel mai defavorabil (deci acurateţe minimă, neliniaritate maximă, eroare maximă la ieşirea de tensiunea) pentru o temperatură de 50 de grade dă o eroare de 0.09-0.1 grade Celsius, în condiţiile în care vorbim de un sistem ieftin, pentru aplicaţii industriale, nu de aparatură de laborator.

Motivul pentru care se folosesc mai multe termometre nu e îmbunătăţirea incertitudinii inerente ci pentru a asigura reducerea incertitudinii de ordine tehnologice -- convertoare analog-numerice care scot bălării pentru că au nimerit un regim tranzitoriu mai lung sau un circuit T&H care a nimerit un caz defavorabil. Ar fi frumos dacă toată treaba s-ar rezuma la a măsura o variaţie de rezistenţă cu un ohmetru sau cu un montaj de punte, dar nu se face aşa :).

Edit: mircea_p, de "apropo"-ul ăla mă fac eu vinovat, dacă te uiţi un pic mai sus cel care a scos gogoriţa cu temperatura medie prima dată am fost eu ;D.

Edit la edit:

CitatÎntr-adevăr media notelor este tot atât de relevantă ca media temperaturilor. Nota 2 plus nota 3 face nota 5 la fel cum 2°C plus 3°C fac 5°C. Si o clasă cu elevi cu IQ 100 este echivalentă cu o clasă în care o treime din elevi au IQ 140 iar două treimi au IQ 80.

Vorbim, din nou, de medierea unor temperaturi din zone similare. Nu adună nimeni temperaturile din Alaska cu cele din Deşertul Kalahari şi face media. Vorbim de creşterea mediilor de temperatură fără modificarea dispersiei zonelor izoterme, adică nu creşte doar media de pe toată harta ci creşte in parte media pe fiecare izotermă. Logic, cu ocazia asta creşte şi media de pe toată harta, dar nu fiindcă în două zone e ca pe Venus în timp ce în restul mai mult se răceşte.

Promit că e ultimul edit ;D ; într-adevăr, temperatura se măsoară cel mai greu dintre unităţile fundamentale, dar incertitudinea depinde de contextul în care faci măsurătoarea. Dacă e vorba de măsurarea pe un domeniu foarte larg (sute de grade sau mai mult, unde se simte abaterea de liniaritate), sau şi mai rău, de măsurarea temperaturii unor amestecuri (nu a unui gaz), o eroare de +/- 2.5-5% e o realizare. Dar măsurarea temperaturii aerului e o chestie foarte convenabilă; domeniul de măsurare e restrâns (nici 100 de grade nu sunt), îţi permiţi să o faci în nişte incinte acceptabil de  bine izolate, staţiile meteo poţi să le pui în fundul pământului şi să le alimentezi din baterii ca să ai o ecranare electromagnetică excelentă -- nu e chiar aşa grav. În asemenea condiţii, 0.05-0.1% pentru tot lanţul de măsurare nu e senzaţional de ambiţios dpdv tehnologic.
Titlu: Măsurarea temperaturii atmosferice și punerea în evidență a schimbărilor climatice
Scris de: tavy din Februarie 01, 2011, 10:03:15 AM
Citat din: mircea_p din Februarie 01, 2011, 12:37:42 AM
Dar ai spus-o foarte bine. Media clasei are o relevanta (sau irelevanta) similara cu media temperaturilor.
La fel cum intr-o clasa cu jumate tampiti si jumate genii poti avea o medie asa si asa, o temperatura medie  (in sensul discutat pana acum) temperata poate insemna niste extreme deosebite de tempratura.
Păi tocmai că respectiva medie este în general irelevantă, o clasa cu jumătate tâmpiți și jumătate genii poate avea cu totul alte realizări în comparație cu o clasă de mediocrii chiar dacă cele două clase au medii egale. Gândeștete, care clasă are mai multe șanse să rezolve o problemă complicată de fizică?

Citat din: mircea_p din Februarie 01, 2011, 12:37:42 AM
Atata timp cat stim ce este media respectiva nu vad nici un pericol in a o folosi. Fie ca o numim temperatura medie sau medie a temperaturilor (stiu, nu sant egale).
In privinta relevantei pentru incalzirea globala, e alta poveste. Nu am studiat destul problema ca sa ma atac.
Ok, să o folosim, pentru ...

Citat din: AlexandruLazar din Februarie 01, 2011, 12:46:18 AM
Un senzor integrat, care vine în capsulă cu tot cu circuitul de condiţionare a semnalului, filtrare şi toate celelalte poate lucra comod la precizii de 0.1% în condiţii departe de cele ideale. N-o zic din aer ci din foaia de catalog: www.analog.com/static/imported-files/data_sheets/3B34.pdf . Cazul cel mai defavorabil (deci acurateţe minimă, neliniaritate maximă, eroare maximă la ieşirea de tensiunea) pentru o temperatură de 50 de grade dă o eroare de 0.09-0.1 grade Celsius, în condiţiile în care vorbim de un sistem ieftin, pentru aplicaţii industriale, nu de aparatură de laborator.
Ok, ai găsit un dispozitiv care măsoară cu oarece precizie o rezistență, maaaare realizare. De la corespondența între rezistența senzorului și până la temperatura senzorului și mai ales de la temperatura senzorului până la temperatura mediului de interes este cale lungă.
Să încerc să mă explic. În general, în fizică, precizia unei măsurători nu este dată numai de aparatele de măsură, există în totdeauna considerente fizice pentru care precizia nu poate fi îmbunătățită indiferent de aparatul care-l folosim. În termodinamică, precizia maximă cu care putem măsura temperatura este dată de fluctuațiile statistice. Aceste fluctuații nu sunt ceva abstract sau subiectiv, ele pot fi calculate suficient de precis. Cu alte cuvinte, putem măsura cu o anumită precizie maximă teoretic temperatura unui gaz într-o incintă cubică cu latura de 1cm și cu o precizie maximă teoretică mult mai mică temperatura aceluiași gaz într-o incintă cubică cu latura de 1m. Această precizie teoretică maximă nu poate fi depășită oricât de precis ar fi instrumentul folosit la măsurătoare.

Citat din: AlexandruLazar din Februarie 01, 2011, 12:46:18 AM
Motivul pentru care se folosesc mai multe termometre nu e îmbunătăţirea incertitudinii inerente ci pentru a asigura reducerea incertitudinii de ordine tehnologice -- convertoare analog-numerice care scot bălării pentru că au nimerit un regim tranzitoriu mai lung sau un circuit T&H care a nimerit un caz defavorabil. Ar fi frumos dacă toată treaba s-ar rezuma la a măsura o variaţie de rezistenţă cu un ohmetru sau cu un montaj de punte, dar nu se face aşa :).
Ai fi surprins cât de precisă poate fi o punte, în special o punte dublă atunci când măsori rezistențe foarte mici.

Citat din: AlexandruLazar din Februarie 01, 2011, 12:46:18 AM
Vorbim, din nou, de medierea unor temperaturi din zone similare. Nu adună nimeni temperaturile din Alaska cu cele din Deşertul Kalahari şi face media. Vorbim de creşterea mediilor de temperatură fără modificarea dispersiei zonelor izoterme, adică nu creşte doar media de pe toată harta ci creşte in parte media pe fiecare izotermă. Logic, cu ocazia asta creşte şi media de pe toată harta, dar nu fiindcă în două zone e ca pe Venus în timp ce în restul mai mult se răceşte.
Nici măcar nu este vorba de temperaturi în zone similare, climatologii vorbesc în mod curent de temperatura medie globală. Chiar și pentru aceeași zonă media temperaturilor înregistrate nu are relevanță fizică și mai ales preciziile alea de 0,1 grade nu au relevanță când dacă măsori temperatura cu astfel de precizii poți obține rezultate diferite la distanțe de numai câțiva metrii în aceeași incintă.

Citat din: AlexandruLazar din Februarie 01, 2011, 12:46:18 AM
într-adevăr, temperatura se măsoară cel mai greu dintre unităţile fundamentale, dar incertitudinea depinde de contextul în care faci măsurătoarea. Dacă e vorba de măsurarea pe un domeniu foarte larg (sute de grade sau mai mult, unde se simte abaterea de liniaritate), sau şi mai rău, de măsurarea temperaturii unor amestecuri (nu a unui gaz), o eroare de +/- 2.5-5% e o realizare. Dar măsurarea temperaturii aerului e o chestie foarte convenabilă; domeniul de măsurare e restrâns (nici 100 de grade nu sunt), îţi permiţi să o faci în nişte incinte acceptabil de  bine izolate, staţiile meteo poţi să le pui în fundul pământului şi să le alimentezi din baterii ca să ai o ecranare electromagnetică excelentă -- nu e chiar aşa grav. În asemenea condiţii, 0.05-0.1% pentru tot lanţul de măsurare nu e senzaţional de ambiţios dpdv tehnologic.
Ce s-ar presupune că trebuie să măsoare stațiile meteo din fundul pământului? Parcă vorbeam de temperatura atmosferei.
Remarc că în mod frecvent vorbești de erori relative în ce privește măsurarea temperaturii, erorile relative au sens în măsurarea temperaturii doar dacă vorbim de o scară absolută (cum ar fi scara Kelvin), în acest caz o eroare de 0,05% pentru temperatura camerei ar fi o eroare de cca. +/- 0.15K.
Mai mult, lanțul de măsurare introduce de obicei erori mult mai mici decât fluctuațiile statistice în cazul măsurătorilor meteo așa că pot să le ignor fără mari probleme.
Vorbești de amestecuri de gaze, îți aduc aminte că aerul este un amestec al naibi de neomogen.

Pentru că am deviat foarte mult de la subiectul treadului ar fi bine ca un administrator să mute mesajele relevante într-un nou tread cu nume de genul ,,Măsurarea temperaturii atmosferice și punerea în evidență a schimbărilor climatice".
Titlu: Măsurarea temperaturii atmosferice și punerea în evidență a schimbărilor climatice
Scris de: AlexandruLazar din Februarie 01, 2011, 01:55:36 PM
CitatOk, ai găsit un dispozitiv care măsoară cu oarece precizie o rezistență, maaaare realizare. De la corespondența între rezistența senzorului și până la temperatura senzorului și mai ales de la temperatura senzorului până la temperatura mediului de interes este cale lungă.

Dispozitivul la care ti-am dat link nu masoara cu precizie o rezistenta, nu e un ohmetru mai destept. Mai arunca un ochi pe datasheet :). Iesirea pe care o scoate e direct in semnal unificat, deci include toata partea de conditionare si liniarizare a semnalului. De acolo il tragi in CAN si pe afisaj, iar daca CAN-ul are cuanta suficient de mica (pentru gama de care vorbim, 12 biti sunt suficienti), nu introduce erori suplimentare. Asa ca acea eroare introdusa de senzor si de conditionare este fix eroarea cu care scoti rezultatul.

CitatNici măcar nu este vorba de temperaturi în zone similare, climatologii vorbesc în mod curent de temperatura medie globală. Chiar și pentru aceeași zonă media temperaturilor înregistrate nu are relevanță fizică și mai ales preciziile alea de 0,1 grade nu au relevanță când dacă măsori temperatura cu astfel de precizii poți obține rezultate diferite la distanțe de numai câțiva metrii în aceeași incintă.

Nu e vorba numai de temperatura medie globala, nimeni nu s-a apucat sa adune cat da la toate izotermele, sa imparta la cate izoterme sunt si sa sara in sus ca e mai cald. E vorba de faptul ca atat media globala variaza, cat si mediile de pe fiecare izoterma. Altfel zis ceea ce acum 50 de ani era o izoterma de 15 grade acum e una de 16 grade sau de 17. Nu e o crestere a mediei tuturor temperaturilor necorelata cu vreo variatie locala, e vorba de faptul ca aproape toate variatiile locale o iau in sus. Asta incerc de cand a inceput tot topicul sa explic, ca nu e o evolutie facuta dintr-o statistica panicarda ci o modificare care are efecte locale evidente.

Statistica globala ar fi fost desigur irelevanta daca efectele locale nu erau corelate. Dar gandeste-te ca nu e vorba ca 51% din banchize se topesc si 49% ingheata, asa ca per ansamblu pare ca toate se topesc alarmant de repede -- e vorba ca *toate* banchizele se topesc.

CitatCe s-ar presupune că trebuie să măsoare stațiile meteo din fundul pământului? Parcă vorbeam de temperatura atmosferei.

Exact, asta incerc si eu sa iti spun. Cand faci o harta izoterma cu o rezolutie standard, nu o sa dai niciodata de exemplul tau cu clasa de genii si clasa de tampiti. Nu o sa ai unul langa altul, la distante de zeci de kilometri, un loc unde sunt 50 de grade si unul unde sunt -40 de grade.

CitatRemarc că în mod frecvent vorbești de erori relative în ce privește măsurarea temperaturii, erorile relative au sens în măsurarea temperaturii doar dacă vorbim de o scară absolută (cum ar fi scara Kelvin), în acest caz o eroare de 0,05% pentru temperatura camerei ar fi o eroare de cca. +/- 0.15K.

Eu remarc ca uiti ca vorbim de foaia de catalog a unui senzor, pe care erorile relative se dau in raport cu domeniul de masurare, nu cu capatul de scara teoretic ;).
Titlu: Măsurarea temperaturii atmosferice și punerea în evidență a schimbărilor climatice
Scris de: mircea_p din Februarie 01, 2011, 04:10:53 PM
Citat din: tavy din Februarie 01, 2011, 10:03:15 AM
Citat din: mircea_p din Februarie 01, 2011, 12:37:42 AM
Dar ai spus-o foarte bine. Media clasei are o relevanta (sau irelevanta) similara cu media temperaturilor.
La fel cum intr-o clasa cu jumate tampiti si jumate genii poti avea o medie asa si asa, o temperatura medie  (in sensul discutat pana acum) temperata poate insemna niste extreme deosebite de tempratura.
Păi tocmai că respectiva medie este în general irelevantă, o clasa cu jumătate tâmpiți și jumătate genii poate avea cu totul alte realizări în comparație cu o clasă de mediocrii chiar dacă cele două clase au medii egale. Gândeștete, care clasă are mai multe șanse să rezolve o problemă complicată de fizică?
Tocmai ca de obicei nu e irelevanta pentru ca e foarte rar sa ai cazul extrem, cu retarzi si genii.
Dar nu asta e punctul pe care il aveam in vedere.

Za zicem ca am o clasa cu 100 de studenti si le dau 3 examene intr-un semestru.
Daca media la primul exam este 40%, la al doilea 60% si la al treilea 70%, insemna ca in mod sigur s-a schimbat ceva de la un examen la altul. Deci media imi spune ca ceva s-a schimbat. Ce anume s-a schimbat nu rezulta direct din medie, nu contest.
Poate ca toti au invatat mai bine, poate ca aia mai slabi au tras mai tare. poate ca unii din cei buni s-au lasat pe tanjeala dara aia slabi au compensat venind din urma puternic. Poate toti au avut nevoie de o perioada de acomodare cu noile concepte sau puri si simplu au gasit o metoda de a trisa la examen.
Media singura nu imi spune care din versiunile astea e corecta dar fara indoiala imi spune ca ceva s-a schimbat.

In general, chestia cu extremele in media unei clase e rara in practica. Profesorii stiu asta si oricand prefera o clasa cu medie mare in anii precedenti. Poti sa-i intrebi.  Sigur ca nu e imposibil ca unul din elevii clasei respective sa fie complet retardat sau sa fie ucigas in serie.


Ca o paranteza:
"Gândeștete, care clasă are mai multe șanse să rezolve o problemă complicată de fizică?"
Nu crezi ca e o intrebare ambigua?
Ce intelegi prin intrebarea asta? Ce inseamna ca "o clasa" sa rezolve problema? Cati din clasa trebuie s-o rezolve ca sa poti spune ca "clasa a rezolvat"? Ori e vorba de a o rezolva "in medie"?
O problema interesanta de fapt. Cred raspunsul ar putea depinde si de gradul de dificultate al problemei.
Poate o formulezi in termenii de genul "care este probabilitatea ca n elevi din clasa respectiva sa rezolve problema.


Titlu: Măsurarea temperaturii atmosferice și punerea în evidență a schimbărilor climatice
Scris de: mircea_p din Februarie 01, 2011, 04:18:55 PM
In privinta discutiei erorilor, cred ca eroarea unui instrument nu e singurul parametru relevant. Eroarea mediei nu e aceeasi cu a instrumentului.
Daca e vorba de cresterea de aproximativ 0.3-0.4 grade care se paote vedea in graficele cu temperatura medie anuala, ar trebui (daca exista dubii) studiata analiza statistica care dovedeste (sau nu) ca aceasta crestere este semnificativa din punct de vedere statistic.
De obicei este considerata "semnificativa" daca probabilitatea de a obtine aceasta variatie din variatii intamplatoare este sub 5%.
Eroarea instrumentelor e doar unul din factori. Eroarea mediei poate fi mai mica decat cea a unei singure masuratori.

Titlu: Măsurarea temperaturii atmosferice și punerea în evidență a schimbărilor climatice
Scris de: AlexandruLazar din Februarie 01, 2011, 04:28:03 PM
Eu mă uit la graficul de aici: http://en.wikipedia.org/wiki/File:2000_Year_Temperature_Comparison.png . Datele sunt din surse respectabile (articole din Nature & co.). Mie nu îmi arată a "hâc" statistic cu nimic mai mult decât perioada de răcire din secolele 14-19, pe care nimeni nu o contestă şi despre care avem dovezi istorice şi geologice că a fost cât se poate de rece din Ţara de Foc şi până-n Groenlanda. Atenţie la axe -- ce e în figură nu sunt mediile temperaturilor.
Titlu: Măsurarea temperaturii atmosferice și punerea în evidență a schimbărilor climatice
Scris de: tavy din Februarie 01, 2011, 05:00:41 PM
Citat din: AlexandruLazar din Februarie 01, 2011, 01:55:36 PM
Dispozitivul la care ti-am dat link nu masoara cu precizie o rezistenta, nu e un ohmetru mai destept. Mai arunca un ochi pe datasheet :). Iesirea pe care o scoate e direct in semnal unificat, deci include toata partea de conditionare si liniarizare a semnalului. De acolo il tragi in CAN si pe afisaj, iar daca CAN-ul are cuanta suficient de mica (pentru gama de care vorbim, 12 biti sunt suficienti), nu introduce erori suplimentare. Asa ca acea eroare introdusa de senzor si de conditionare este fix eroarea cu care scoti rezultatul.
Ok. Admit că dispozitivul dat de tine face analogic oarece corecții care se puteau face oricum digital, irelevant, pentru că nu mă interesează cât de ușor ajunge valoarea măsurată pe afișaj.
Până la urmă tot temperatura senzorului o măsori cu precizie, temperatura atmosferei nu poți să o măsori cu aceiași precizie din cauza fluctuațiilor statistice.

Citat din: AlexandruLazar din Februarie 01, 2011, 01:55:36 PM
Nu e vorba numai de temperatura medie globala, nimeni nu s-a apucat sa adune cat da la toate izotermele, sa imparta la cate izoterme sunt si sa sara in sus ca e mai cald. E vorba de faptul ca atat media globala variaza, cat si mediile de pe fiecare izoterma. Altfel zis ceea ce acum 50 de ani era o izoterma de 15 grade acum e una de 16 grade sau de 17. Nu e o crestere a mediei tuturor temperaturilor necorelata cu vreo variatie locala, e vorba de faptul ca aproape toate variatiile locale o iau in sus. Asta incerc de cand a inceput tot topicul sa explic, ca nu e o evolutie facuta dintr-o statistica panicarda ci o modificare care are efecte locale evidente.

Statistica globala ar fi fost desigur irelevanta daca efectele locale nu erau corelate. Dar gandeste-te ca nu e vorba ca 51% din banchize se topesc si 49% ingheata, asa ca per ansamblu pare ca toate se topesc alarmant de repede -- e vorba ca *toate* banchizele se topesc.
Bănuiesc că prin topirea banchizelor te referi la reducerea suprafeței/volumului lor. Din cunoștințele mele dimensiunea banchizei variază în funcție de cantitatea de ghiață pierdută prin topire/rupere/sublimare și cantitatea de ghiață primită din precipitații. Intuitiv, dacă am avea o ,,răcire globală" s-ar evapora mai puțină apă din oceane și m-aș aștepta ca și volumul precipitațiilor să scadă. Cu alte cuvinte dimensiunea banchizelor poate să scadă și din cauza ,,răcirii globale". Oricum, în știință, corelația nu implică cauzalitatea.

Încă nu mi-a dat nimeni nici un exemplu de formulă din fizică în care să se facă operații de adunare între temperaturi. Extind puțin, poate cineva să-mi dea un singur exemplu din fizică în care se adună sau se face media, orice medie, a unor mărimi intensive, nu neapărat temperatură?

Citat din: mircea_p din Februarie 01, 2011, 04:10:53 PM
Ca o paranteza:
"Gândeștete, care clasă are mai multe șanse să rezolve o problemă complicată de fizică?"
Nu crezi ca e o intrebare ambigua?
Ce intelegi prin intrebarea asta? Ce inseamna ca "o clasa" sa rezolve problema? Cati din clasa trebuie s-o rezolve ca sa poti spune ca "clasa a rezolvat"? Ori e vorba de a o rezolva "in medie"?
O problema interesanta de fapt. Cred raspunsul ar putea depinde si de gradul de dificultate al problemei.
Poate o formulezi in termenii de genul "care este probabilitatea ca n elevi din clasa respectiva sa rezolve problema.
Problema era cât se poate de clară, problema trebuie să o rezolve clasa, nu mă interesează în ce mod, poate să o rezolve unul singur din clasă si ceilalți să se uite la el sau pot să colaboreze toți la rezolvare.

Citat din: mircea_p din Februarie 01, 2011, 04:18:55 PM
Eroarea instrumentelor e doar unul din factori. Eroarea mediei poate fi mai mica decat cea a unei singure masuratori.
Asta nu este adevărat. De unde ai scos-o, o formulă ceva?

Citat din: AlexandruLazar din Februarie 01, 2011, 04:28:03 PM
Eu mă uit la graficul de aici: http://en.wikipedia.org/wiki/File:2000_Year_Temperature_Comparison.png . Datele sunt din surse respectabile (articole din Nature & co.). Mie nu îmi arată a "hâc" statistic cu nimic mai mult decât perioada de răcire din secolele 14-19, pe care nimeni nu o contestă şi despre care avem dovezi istorice şi geologice că a fost cât se poate de rece din Ţara de Foc şi până-n Groenlanda. Atenţie la axe -- ce e în figură nu sunt mediile temperaturilor.
O sursă care face media temperaturilor nu este credibilă din start.
Citat
The temperature anomaly is referred to the NCDC (National Climate Data Centre)'s calculation of the mean temperature of land and ocean by collecting the data from thousands of observation sites worldwide. It can be calculated the global anomalies temperature covering the inaccessible areas such as top of mountains, far away sea surfaces, and desert areas.

Și dacă tot dam linkuri spre articole: http://www.sciencedaily.com/releases/2007/03/070315101129.htm (http://www.sciencedaily.com/releases/2007/03/070315101129.htm)
Titlu: Re: Măsurarea temperaturii atmosferice și punerea în evidență a schimbărilor climatice
Scris de: Electron din Februarie 01, 2011, 10:50:40 PM
Citat din: tavy din Februarie 01, 2011, 10:03:15 AM
Pentru că am deviat foarte mult de la subiectul treadului ar fi bine ca un administrator să mute mesajele relevante într-un nou tread cu nume de genul ,,Măsurarea temperaturii atmosferice și punerea în evidență a schimbărilor climatice".
Am despartit discutia de fata de topicul original.

Acuma poate decide Adi in ce sectiune ar trebui mutata aceasta discutie...

e-
Titlu: Re: Măsurarea temperaturii atmosferice și punerea în evidență a schimbărilor climatice
Scris de: AlexandruLazar din Februarie 01, 2011, 11:40:27 PM
"Temperatura" globală e departe de a fi singurul indicator. Uite aici cum altcumva se mai face dacă tot vorbeam de NASA: http://data.giss.nasa.gov/gistemp/, şi găseşti pe acolo şi ceva dataset-uri ca să vezi cu e cu evoluţia temperaturilor, inclusiv local.
Titlu: Re: Măsurarea temperaturii atmosferice și punerea în evidență a schimbărilor climatice
Scris de: tavy din Februarie 02, 2011, 12:05:17 AM
Citat din: AlexandruLazar din Februarie 01, 2011, 11:40:27 PM
"Temperatura" globală e departe de a fi singurul indicator. Uite aici cum altcumva se mai face dacă tot vorbeam de NASA: http://data.giss.nasa.gov/gistemp/, şi găseşti pe acolo şi ceva dataset-uri ca să vezi cu e cu evoluţia temperaturilor, inclusiv local.
Chiar nu am acum nici timpul și nici răbdarea să citesc încă un articol în care se tot vorbește despre ,,temperature anomalies" și ,,global temperature", am citit mult prea multe articole pe tema asta ca să-mi fac o idee.
Oricum apelul la autoritate care-l încerci tu aici nu este tocmai un argument valid într-o discuție, fluctuațiile statistice există independent de opinia NASA, temperatura tot mărime intensivă rămâne, atmosfera continuă să fie un sistem haotic inpredictibil iar gradientul temperaturii în atmosferă rămâne suficient de mare ca măsurările pe un grid foarte mare să fie irelevante pentru modelarea evoluției sistemului.
http://ro.wikipedia.org/wiki/Apelul_la_autoritate (http://ro.wikipedia.org/wiki/Apelul_la_autoritate)
Titlu: Re: Măsurarea temperaturii atmosferice și punerea în evidență a schimbărilor climatice
Scris de: AlexandruLazar din Februarie 02, 2011, 12:16:14 AM
Nu e un apel la autoritate, încerc să îţi arăt care e metodologia de lucru. Dacă de fiecare dată când cineva a făcut asta ai înţeles-o ca pe un apel la autoritate nici nu e de mirare că nu ştii de fapt cum se fac observaţiile de tipul ăsta.

Nu se lucrează, pentru modele climatice, cu medii de temperaturi de pe tot globul, ceea ce n-ar avea nicio logică întrucât nu e aceeaşi zonă de climă peste tot. Se lucrează medii făcute local iar studiul evoluţiei se face tot pe zone, de exemplu ca aici:

http://data.giss.nasa.gov/cgi-bin/gistemp/do_nmap.py?year_last=2010&month_last=12&sat=4&sst=1&type=trends&mean_gen=12&year1=1950&year2=2010&base1=1951&base2=1980&radius=1200&pol=reg

Este evolutia temperaturilor înregistrate din satelit din 1950 până azi -- e doar o hartă şi un grafic, ai timp să te uiţi. O să vezi că peste tot, temperaturile înregistrate *local* au crescut peste tot, numai în 5-6 zone, foarte mici, de pe toată harta, majoritatea deasupra oceanelor sau în zonele de turnură a curenţilor marini reci, s-au produs răciri, şi acelea cu temperaturi modeste.

Deci nu e nicio clasă unde sunt genii şi retardaţi.
Titlu: Re: Măsurarea temperaturii atmosferice și punerea în evidență a schimbărilor climatice
Scris de: tavy din Februarie 02, 2011, 12:43:02 AM
Citat din: AlexandruLazar din Februarie 02, 2011, 12:16:14 AM
Nu se lucrează, pentru modele climatice, cu medii de temperaturi de pe tot globul, ceea ce n-ar avea nicio logică întrucât nu e aceeaşi zonă de climă peste tot. Se lucrează medii făcute local iar studiul evoluţiei se face tot pe zone, de exemplu ca aici:

http://data.giss.nasa.gov/cgi-bin/gistemp/do_nmap.py?year_last=2010&month_last=12&sat=4&sst=1&type=trends&mean_gen=12&year1=1950&year2=2010&base1=1951&base2=1980&radius=1200&pol=reg
Să înțeleg că dacă se ia în considerare temperatura doar local aceasta devine mărime extensivă?

Citat din: AlexandruLazar din Februarie 02, 2011, 12:16:14 AM
Este evolutia temperaturilor înregistrate din satelit din 1950 până azi -- e doar o hartă şi un grafic, ai timp să te uiţi. O să vezi că peste tot, temperaturile înregistrate *local* au crescut peste tot, numai în 5-6 zone, foarte mici, de pe toată harta, majoritatea deasupra oceanelor sau în zonele de turnură a curenţilor marini reci, s-au produs răciri, şi acelea cu temperaturi modeste.

Deci nu e nicio clasă unde sunt genii şi retardaţi.
De ce 1950 și nu 1955 sau orice alt an, ce are special 1950?
Temperaturi măsurate din satelit în 1950?  ??? Uau.
Chiar și așa, imaginează-ti harta respectivă înfășurată pe un glob, am impresia că harta folosește o proiecție cilindrică care te păcălește foarte ușor în ce privește suprafața la poli.  Dacă faci asta îți dai seama imediat că marea pată roșie de la polul nord este mult mai mică ca suprafață decât pare. Apoi ignoră fără probleme variațiile mai mici de +/-1 grad pentru că sunt irelevante din punct de vedere al măsurătorii, oare de ce nu au folosit o scară liniară de culori și au pus mai multe culori în zona variațiilor mici de temperatură? Înlocuiește deci bleu, vernil, galben și portocaliu deschis (ce este între -1 și 1) cu alb.
După ce faci asta vei putea remarca ușor că ce rămâne pot fi numite, cel mult, fenomene locale de încălzire/răcire, nicidecum un fenomen global. Și am fost generos eliminând doar variațiile de +/- 1 grad.
Titlu: Re: Măsurarea temperaturii atmosferice și punerea în evidență a schimbărilor climatice
Scris de: AlexandruLazar din Februarie 02, 2011, 12:56:33 AM
CitatDe ce 1950 și nu 1955 sau orice alt an, ce are special 1950?

Nimic, era cel mai vechi an pe care îl aveau acolo. Uite din 1970 dacă vrei, de când au început scanările GISS: http://data.giss.nasa.gov/cgi-bin/gistemp/do_nmap.py?year_last=2010&month_last=12&sat=4&sst=1&type=trends&mean_gen=12&year1=1970&year2=2010&base1=1970&base2=1980&radius=1200&pol=reg

CitatSă înțeleg că dacă se ia în considerare temperatura doar local aceasta devine mărime extensivă?

Deja ai pretenţii absurde. Ok: media făcută cu temperaturile de pe tot globul este puţin relevantă pentru comportamentul local pe fiecare zonă în parte, lucru cu care dealtfel sunt perfect de acord. Dar vrei să spui că mediile făcute local sunt irelevante pentru comportamentul local? Totuşi nu vorbim aici de aerul dintr-un compresor! Ţi s-a întâmplat vreodată ca într-o săptămână să fie 3 zile cu -20 de grade şi 4 cu +30?

Edit: că mi-am adus aminte:

CitatOk. Admit că dispozitivul dat de tine face analogic oarece corecții care se puteau face oricum digital, irelevant, pentru că nu mă interesează cât de ușor ajunge valoarea măsurată pe afișaj.

Serios? Şi ai încredere în corecţiile şi valorile obţinute digital?
Nu de alta dar dacă e aşa, ar fi interesant de amintit că temperatura pe care o obţii la sfârşit, obţinută din date eşantionate, e de fapt rezultatul unor medieri a datelor obţinute la eşantionare, nu pentru că le place lor ci pentru că aşa lucrează un convertor analog-numeric, iar intrarea e oricum filtrată pentru a elimina zgomotele de bandă largă. Ceea ce vezi pe ecranul unui termometru digital e o medie de temperaturi ;).
Titlu: Re: Măsurarea temperaturii atmosferice și punerea în evidență a schimbărilor climatice
Scris de: Mihnea Maftei din Februarie 02, 2011, 02:05:24 AM
Citat din: tavy din Februarie 01, 2011, 05:00:41 PM
Citat din: mircea_p din Februarie 01, 2011, 04:18:55 PM
Eroarea instrumentelor e doar unul din factori. Eroarea mediei poate fi mai mica decat cea a unei singure masuratori.
Asta nu este adevărat. De unde ai scos-o, o formulă ceva?

Ba da, e adevarat. Erorile sunt, prin natura lor, de obicei aleatorii si au o medie aproape de zero. Pentru un numar mare de masuratori, eroarea mediei poate fi extrem de mica.

Tavy, inteleg ca scepticismul tau se bazeaza in principal pe doua lucruri:
1. temperatura se masoara foarte imprecis (si o crestere masurata a temperaturii de 0,3 grade C este irelevanta)
2. faptul ca temperatura e o marime intensiva (si, prin urmare, o medie de temperaturi e o notiune fara sens)

Legat de 1., nu ai dreptate, mai ales pentru ca erorile de masurare dispar in mare masura prin mediere. La fel si erorile cauzate de faptul ca nu exista statii meteo in fiecare centimetru patrat de pe Pamant.

Legat de 2., nu e un argument destul de puternic in problema asta. Gandeste-te ca ai fi intr-o casa fara ferestre si nu ai sti ce anotimp e. Ai avea acces doar la media a zece temperaturi din tara, in fiecare zi. Ai putea detecta daca e vara sau iarna?
Titlu: Re: Măsurarea temperaturii atmosferice și punerea în evidență a schimbărilor climatice
Scris de: AlexandruLazar din Februarie 02, 2011, 02:34:45 AM
Legat de erori -- poate n-ar strica să ne amintim câte ceva despre ele ;).

Cele care nu se compensează prin mediere sunt erorile sistematice, care apar la fiecare măsurare cam cu aceeaşi magnitudine şi cu acelaşi semn. De exemplu, dacă pentru măsurare se foloseşte un senzor Pt100 care are o eroare de offset de 1 ohm, şi la 0 grade are 101 ohmi în loc de 100, cinci măsurători n-or să amelioreze cu nimic rezultatul pentru că, în medie, tot 101 ohmi o să iasă că are.

Cele care se compensează prin mediere sunt erorile nesistematice, care nu apar mereu cu acelaşi semn şi magnitudine. De exemplu, erorile de neliniaritate datorate neomogenităţii materialului din care e făcut senzorul. De exemplu, dacă ai 4 senzori, temperatura reală e de 0 grade şi toţi senzorii ar trebui să dea (să zicem) 100 de ohmi, dar cei 4 senzori au 100.2, 99.8, 100.1, 100.1 ohmi, prin mediere se obţine 100.05 ohmi, care e de fapt mai aproape de rezultatul de 100 de ohmi decât oricare din cele 4 indicaţii. Ca fapt divers, de aceea modulele pentru sisteme de măsurare distribuite sunt adesea fabricate din componente provenind din loturi de fabricaţie diferite, astfel încât influenţele obţinute tehnologic să nu fie toate de acelaşi fel.

Cel puţin în ceea ce priveşte senzorii de tip RTD, erorile de offset nu sunt mari pentru că procesul de fabricaţie permite controlul lor într-un mod acceptabil. Pentru senzorii de temperaturi mari, cu fel şi fel de joncţiuni, e ceva mai greu.

Terminologia legală cred că e de erori de justeţe pentru cele care nu se compensează prin mediere şi de erori de fidelitate pentru cele care se compensează, dar nu le mai ştiu acum pe dinafară la câte zeci de feluri de erori au inventat ăştia ;D.
Titlu: Re: Măsurarea temperaturii atmosferice și punerea în evidență a schimbărilor climatice
Scris de: Adi din Februarie 02, 2011, 03:02:48 AM
Citat din: AlexandruLazar din Februarie 02, 2011, 02:34:45 AM
Terminologia legală cred că e de erori de justeţe pentru cele care nu se compensează prin mediere şi de erori de fidelitate pentru cele care se compensează, dar nu le mai ştiu acum pe dinafară la câte zeci de feluri de erori au inventat ăştia ;D.

In fizica particulelor noi zicem erori sistematice in loc de erori de justete si erori statistice in loc de erori de fidelitate. Astea sunt cele doua mari tipuri de erori in general din stiinta, dupa cate stiu eu.
Titlu: Re: Măsurarea temperaturii atmosferice și punerea în evidență a schimbărilor climatice
Scris de: mircea_p din Februarie 02, 2011, 07:15:54 AM
Citat din: tavy din Februarie 01, 2011, 05:00:41 PM
Încă nu mi-a dat nimeni nici un exemplu de formulă din fizică în care să se facă operații de adunare între temperaturi. Extind puțin, poate cineva să-mi dea un singur exemplu din fizică în care se adună sau se face media, orice medie, a unor mărimi intensive, nu neapărat temperatură?
Obstinatia asta cu fizica si parametrii intensivi aduce cumva cu faimoasa argumentare de tipul "om de paie" (atribui altora afirmatii pe care nu le-au facut si apoi le combati) . A zis cineva ca mediile de temepratura folosite in meteorologie sant acelasi lucru cu temperatura termodinamica?

Mi se pare ca problema aici este care este semnificatia modificarilor in temperatura medie masurate de climatologi si daca se pot sau nu trage concluzii in privinta evolutiei viitoare.
Chiar daca ceea ce masoara ei nu apare in vreo formula de fizica, nu inseamna ca nu exista. Cel putin nu in mod necesar.

Poate ar fi mai util sa expui punctul propriu de vedere asupra schimbarilor climatice.
Crezi ca nu exista de loc? Sau sant modificari locale care nu corespund unora "globale"? Sau sant modificari, dar se incadreaza in variatiile "normale"?
Astea sant intrebari care nu se refera la posibile cauze sau la anumiite teorii ci doar daca exista sau nu un efect observabil.


Titlu: Re: Măsurarea temperaturii atmosferice și punerea în evidență a schimbărilor climatice
Scris de: tavy din Februarie 02, 2011, 09:56:40 AM
Citat din: AlexandruLazar din Februarie 02, 2011, 12:56:33 AM
Serios? Şi ai încredere în corecţiile şi valorile obţinute digital?
Nu de alta dar dacă e aşa, ar fi interesant de amintit că temperatura pe care o obţii la sfârşit, obţinută din date eşantionate, e de fapt rezultatul unor medieri a datelor obţinute la eşantionare, nu pentru că le place lor ci pentru că aşa lucrează un convertor analog-numeric, iar intrarea e oricum filtrată pentru a elimina zgomotele de bandă largă. Ceea ce vezi pe ecranul unui termometru digital e o medie de temperaturi ;).
Ești sigur că afișezi o medie de temperaturi și nu media altei mărimi fizice care medie este funcție de temperatură și nu funcție de media temperaturilor?

Citat din: Mihnea din Februarie 02, 2011, 02:05:24 AM
Ba da, e adevarat. Erorile sunt, prin natura lor, de obicei aleatorii si au o medie aproape de zero. Pentru un numar mare de masuratori, eroarea mediei poate fi extrem de mica.

Tavy, inteleg ca scepticismul tau se bazeaza in principal pe doua lucruri:
1. temperatura se masoara foarte imprecis (si o crestere masurata a temperaturii de 0,3 grade C este irelevanta)
2. faptul ca temperatura e o marime intensiva (si, prin urmare, o medie de temperaturi e o notiune fara sens)

Legat de 1., nu ai dreptate, mai ales pentru ca erorile de masurare dispar in mare masura prin mediere. La fel si erorile cauzate de faptul ca nu exista statii meteo in fiecare centimetru patrat de pe Pamant.

Legat de 2., nu e un argument destul de puternic in problema asta. Gandeste-te ca ai fi intr-o casa fara ferestre si nu ai sti ce anotimp e. Ai avea acces doar la media a zece temperaturi din tara, in fiecare zi. Ai putea detecta daca e vara sau iarna?
Legat de 1.: Erorile de măsură dispar în foarte mică măsură prin mediere.
Să facem un calcul:
Dacă avem masurătorile:
[tex]T_1, T_2, \dots, T_n[/tex]
Masurate cu erorile absolute:
[tex]\delta T_1, \delta T_2 \dots, \delta T_n[/tex]
Media va fi:
[tex]\overline{T} = \frac{\sum_{i=1}^{n} T_i} {n}[/tex]
Iar eroarea absolută va fi:
[tex]\delta\overline{T} = \frac{\sum_{i=1}^{n} \delta T_i} {n}[/tex]
Dacă presupunem că toate masurătorile au fost făcute cu aceeași eroare relativă [tex]\delta T[/tex] atunci eroarea absolută a mediei va fi tot [tex]\delta T[/tex], poți observa că prin mediere nu am obținut o imbunătățire a erorii de măsură.

Legat de 2: Pot să am acces de exemplu și la maximele sau minimele temperaturilor sau la temperatura măsurată la aceeași oră zilnic și tot pot detecta dacă este iarnă sau vară, media nu-mi aduce nici o informație în plus.

Citat din: AlexandruLazar din Februarie 02, 2011, 02:34:45 AM
De exemplu, dacă ai 4 senzori, temperatura reală e de 0 grade şi toţi senzorii ar trebui să dea (să zicem) 100 de ohmi, dar cei 4 senzori au 100.2, 99.8, 100.1, 100.1 ohmi, prin mediere se obţine 100.05 ohmi, care e de fapt mai aproape de rezultatul de 100 de ohmi decât oricare din cele 4 indicaţii. Ca fapt divers, de aceea modulele pentru sisteme de măsurare distribuite sunt adesea fabricate din componente provenind din loturi de fabricaţie diferite, astfel încât influenţele obţinute tehnologic să nu fie toate de acelaşi fel.
Dacă cei 4 senzori îi masori cu o cifră semnificativă după virgulă, ceea ce duce la ideea că faci o măsuratoare cu precizie de [tex]0.1\Omega[/tex] și obții rezultatele date de tine cred că trebuie să te uiți unde ai greșit și să repeți măsurătoarea.
Oricum aici nu era vorba de precizia aparatului de măsură ci de precizia maximă cu care poți măsura temperatura unui sistem, presupunând că ai aparatul de măsură ideal. În cazul atmosferei precizia maximă teoretică nu atinge valori atât de mari.

Citat din: mircea_p din Februarie 02, 2011, 07:15:54 AM
Obstinatia asta cu fizica si parametrii intensivi aduce cumva cu faimoasa argumentare de tipul "om de paie" (atribui altora afirmatii pe care nu le-au facut si apoi le combati) . A zis cineva ca mediile de temepratura folosite in meteorologie sant acelasi lucru cu temperatura termodinamica?
Nu trebuie să fie același lucru cu temperatura termodinamică, trebuie doar să aibă o semnificație fizică pentru că altfel nu poate fi folosită decât în numerologie climatologie.

Citat din: mircea_p din Februarie 02, 2011, 07:15:54 AM
Mi se pare ca problema aici este care este semnificatia modificarilor in temperatura medie masurate de climatologi si daca se pot sau nu trage concluzii in privinta evolutiei viitoare.
Chiar daca ceea ce masoara ei nu apare in vreo formula de fizica, nu inseamna ca nu exista. Cel putin nu in mod necesar.
Chiar dacă nu putem detecta inorogul roz indetectabil care trăiește în centrul pământului, nu înseamnă că nu există.

Citat din: mircea_p din Februarie 02, 2011, 07:15:54 AM
Poate ar fi mai util sa expui punctul propriu de vedere asupra schimbarilor climatice.
Crezi ca nu exista de loc? Sau sant modificari locale care nu corespund unora "globale"? Sau sant modificari, dar se incadreaza in variatiile "normale"?
Astea sant intrebari care nu se refera la posibile cauze sau la anumiite teorii ci doar daca exista sau nu un efect observabil.
Nu susțin că nu există schimbări climatice.
Schimbările climatice pot fi și globale, nici o problemă.
Nu știu ce înseamnă ,,variații normale" deci nu mă pot pronunța cu privere la încadrarea în ,,variațiile normale".
În mod subiectiv, de data aceasta, consider că schimbările climatice, din această perioadă istorică, nu sunt deloc spactaculoase și sunt oricum mult mai mici decât altele petrecute în istorie, fără a lua în calcul și preistoria.
În ce privește schimbarile climatice globale influențate de activitatea umană sunt foarte sceptic, nu a reușit  nimeni să-mi vină cu argumente științifice prin care să-mi demonstreze că activitatea umană ar avea o influență, oricât de mică dar detectabilă, asupra climei la nivel global.
Deasemenea nu sunt de acord nu felul în care sunt studiate schimbările climatice sau cel puțin nu sunt de acord cu modul în care sunt prezentate publicului aceste studii.
Sunt convins că modelele prin care se fac prognoze pe termen relativ lung 10 ani, 1000 ani sunt irelevante în stadiul actual de dezvoltare a tehnologiei. Chestii de genul temperatura va crește/scade cu [tex]n[/tex] grade în următorii [tex]x[/tex] ani unde [tex]x>1 an[/tex] mi se par fabulații lipsite de orice bază științifică.
Titlu: Re: Măsurarea temperaturii atmosferice și punerea în evidență a schimbărilor climatice
Scris de: AlexandruLazar din Februarie 02, 2011, 02:17:31 PM
Citat din: tavy din Februarie 02, 2011, 09:56:40 AM
Citat din: AlexandruLazar din Februarie 02, 2011, 12:56:33 AM
Serios? Şi ai încredere în corecţiile şi valorile obţinute digital?
Nu de alta dar dacă e aşa, ar fi interesant de amintit că temperatura pe care o obţii la sfârşit, obţinută din date eşantionate, e de fapt rezultatul unor medieri a datelor obţinute la eşantionare, nu pentru că le place lor ci pentru că aşa lucrează un convertor analog-numeric, iar intrarea e oricum filtrată pentru a elimina zgomotele de bandă largă. Ceea ce vezi pe ecranul unui termometru digital e o medie de temperaturi ;).
Ești sigur că afișezi o medie de temperaturi și nu media altei mărimi fizice care medie este funcție de temperatură și nu funcție de media temperaturilor?

Foarte sigur :). In cazul statiilor meteo nu e o chestie foarte evidenta -- datele care acolo se obtin prin mediere sunt rezultatele inregistrate in timp (graficele de temperatura) pentru ca datele din ele se obtin prin supraesantionare iar reconstituirea datelor analogice se face printr-un filtru de netezire, care practic face o mediere pe perioada de esantionare, sau prin interpolare liniara care porneste inerent prin a face o mediere ca sa obtina panta. Propriu-zis si esantionarea presupune o mediere, dar acolo sa zicem ca nu e o problema pentru ca temperatura este atat de lent variabila incat medierea (care se face analogic) nu afecteaza decat fluctuatiile datorate zgomotului. Deci, in statiile meteo, valorile individuale sunt ok; graficele de temperatura in schimb nu.

In schimb la termometrele digitale de panou sau de buzunar temperatura pe care o vezi e mediata "pe fata". Astea nu lucreaza cu senzori Pt100 sau alte asemenea, pentru ca au sensibilitate prea mica, folosesc fie termistori liniari fie cel mai adesea astazi, senzori integrati cu jonctiune. Ca sa nu complice partea analogica sau convertoarele de masurare, eliminarea zgomotelor si a fluctuatiilor interne se face direct pe microcontrollerul integrat, printr-un filtru medie alunecatoare de ordin 4 sau 8. Ca sa nu stoarca bateria lucrand prea repede, cele 4 sau 8 esantioane se preleveaza la intervale destul de mari unul de altul, deci ceea ce vezi la orice moment pe ele e media a 4 sau 8 temperaturi luate in ultimele 20-30 de secunde.

CitatDacă presupunem că toate masurătorile au fost făcute cu aceeași eroare relativă  atunci eroarea absolută a mediei va fi tot , poți observa că prin mediere nu am obținut o imbunătățire a erorii de măsură.

Hehe, tocmai :). Nu toate componentele erorii relative se regăsesc la fel de fiecare dată. Într-un lanţ de măsurare a temperaturii, erorile pe care le întâlneşti mereu la fel vor fi erorile de offset ale senzorului şi amplificatorului de intrare, erorile de pantă ale senzorului şi amplificatorului de intrare şi deriva termică a componentelor integrate.

Erorile de offset ale senzorului şi ale amplificatorului se pot compensa -- la senzor prin calibrare, la amplificatoarele de intrare se poate compensa automat de către amplificator, din construcţie. Erorile de pantă sunt foarte scăzute pentru că lucrul se face pe o zonă foarte restrânsă a domeniului de lucru; fluctuaţia faţă de temperatura de referinţă la senzor sunt mici (30-40 de grade în fiecare direcţie), iar tensiunea de intrare furnizată amplificatorului e şi ea mică, până în nişte sute de mV, aşa că erorile de pantă nu se simt mai deloc. Deriva termică e cea mai greu de compensat dar ea nu intervine decât la măsurarea în condiţii extreme -- pe +50 de grade sau -30 de grade, caz în care de regulă se rezolvă prin termostatarea montajelor.

CitatDacă cei 4 senzori îi masori cu o cifră semnificativă după virgulă, ceea ce duce la ideea că faci o măsuratoare cu precizie de  și obții rezultatele date de tine cred că trebuie să te uiți unde ai greșit și să repeți măsurătoarea.
Oricum aici nu era vorba de precizia aparatului de măsură ci de precizia maximă cu care poți măsura temperatura unui sistem, presupunând că ai aparatul de măsură ideal. În cazul atmosferei precizia maximă teoretică nu atinge valori atât de mari.

Tocmai, presupuneam că nu vorbim de precizia aparatului de măsură; dar dacă insişti să introducem şi aparatul de măsură, limitând precizia la o zecimală, n-am nimic împotrivă, nu de alta dar întrucât rotunjirea în astfel de sisteme se face de fapt prin trunchiere, rezultă la mediere 100 de ohmi în cap :).

CitatNu susțin că nu există schimbări climatice.
Schimbările climatice pot fi și globale, nici o problemă.
Nu știu ce înseamnă ,,variații normale" deci nu mă pot pronunța cu privere la încadrarea în ,,variațiile normale".
În mod subiectiv, de data aceasta, consider că schimbările climatice, din această perioadă istorică, nu sunt deloc spactaculoase și sunt oricum mult mai mici decât altele petrecute în istorie, fără a lua în calcul și preistoria.
În ce privește schimbarile climatice globale influențate de activitatea umană sunt foarte sceptic, nu a reușit  nimeni să-mi vină cu argumente științifice prin care să-mi demonstreze că activitatea umană ar avea o influență, oricât de mică dar detectabilă, asupra climei la nivel global.
Deasemenea nu sunt de acord nu felul în care sunt studiate schimbările climatice sau cel puțin nu sunt de acord cu modul în care sunt prezentate publicului aceste studii.
Sunt convins că modelele prin care se fac prognoze pe termen relativ lung 10 ani, 1000 ani sunt irelevante în stadiul actual de dezvoltare a tehnologiei. Chestii de genul temperatura va crește/scade cu  grade în următorii  ani unde  mi se par fabulații lipsite de orice bază științifică.

Acuma ca fapt divers -- cu o mare parte din poziţia asta sunt şi eu de acord. Schimbarea actuală nu e cu nimic mai mare decât vârful de încălzire medieval, nici decât răcirea din secolele 14-18, care n-au avut nici pe departe efecte catastrofale. Corelaţia nu înseamnă automat implicaţie, deci magnitudinea rolului jucat de activitatea antropică e greu de cuantificat, şi dat fiind stadiul actual al modelelor climatice, predicţii pe mai mult de 1 an sunt greu de făcut. Hell, ăştia nu nimeresc de fiecare dată starea vremii peste 3 zile.
Titlu: Re: Măsurarea temperaturii atmosferice și punerea în evidență a schimbărilor climatice
Scris de: tavy din Februarie 02, 2011, 04:58:21 PM
Citat din: AlexandruLazar din Februarie 02, 2011, 02:17:31 PM
Citat din: tavy din Februarie 02, 2011, 09:56:40 AM
Citat din: AlexandruLazar din Februarie 02, 2011, 12:56:33 AM
Serios? Şi ai încredere în corecţiile şi valorile obţinute digital?
Nu de alta dar dacă e aşa, ar fi interesant de amintit că temperatura pe care o obţii la sfârşit, obţinută din date eşantionate, e de fapt rezultatul unor medieri a datelor obţinute la eşantionare, nu pentru că le place lor ci pentru că aşa lucrează un convertor analog-numeric, iar intrarea e oricum filtrată pentru a elimina zgomotele de bandă largă. Ceea ce vezi pe ecranul unui termometru digital e o medie de temperaturi ;).
Ești sigur că afișezi o medie de temperaturi și nu media altei mărimi fizice care medie este funcție de temperatură și nu funcție de media temperaturilor?

Foarte sigur :).
Lanțul tău de măsură este format dintr-un senzor care transformă temperatura într-o marime electrică, tensiune, curent, rezistență nu are importanță.
Mai apoi mărimea electrică o aplici unui convertor A/D, în general înainte de convertor ai un circuit care reține valoarea mărimii electrice pe parcursul măsurării, avea un nume dar îmi scapă acum pentru că nu am mai lucrat de mult. Convertorul A/D poate fi cu integrare în timp (poate greșesc termeni dar înțelegi tu) sau poate avea un comparator un convertor D/A și un registru cu aproximații succesive, nu cred că aici se aplică alte variante decât astea ținând cont că vrem cel puțin 10 bit. Undeva în lanț, cât timp avem mărimea electrică vor apare filtre pentru eliminarea zgomotelor.
Acum gândeștete unde în lanțul acesta apare medierea și ne putem da seama ce mărime se mediază.

Citat din: AlexandruLazar din Februarie 02, 2011, 02:17:31 PM
CitatDacă presupunem că toate masurătorile au fost făcute cu aceeași eroare relativă  atunci eroarea absolută a mediei va fi tot , poți observa că prin mediere nu am obținut o imbunătățire a erorii de măsură.

Hehe, tocmai :). Nu toate componentele erorii relative se regăsesc la fel de fiecare dată. Într-un lanţ de măsurare a temperaturii, erorile pe care le întâlneşti mereu la fel vor fi erorile de offset ale senzorului şi amplificatorului de intrare, erorile de pantă ale senzorului şi amplificatorului de intrare şi deriva termică a componentelor integrate.
Când m-am referit la erori m-am referit la ele ca un fel de invers al preciziei, eroarea exactă nu o știm pentru că ar trebui să facem diferența între valoarea măsurată și valoarea reală, dacă am ști valoarea reală nu am mai avea nevoie să facem măsurătoarea.
Cu alte cuvinte, dacă măsor o lungime cu o riglă cu gradația la 1mm pot spune că am măsurat cu o eroare de 1mm (dacă ignor alte surse de erori care pot fi neglijabile dacă măsor ceva destul de mic).

Citat din: AlexandruLazar din Februarie 02, 2011, 02:17:31 PM
CitatNu susțin că nu există schimbări climatice.
Schimbările climatice pot fi și globale, nici o problemă.
Nu știu ce înseamnă ,,variații normale" deci nu mă pot pronunța cu privere la încadrarea în ,,variațiile normale".
În mod subiectiv, de data aceasta, consider că schimbările climatice, din această perioadă istorică, nu sunt deloc spactaculoase și sunt oricum mult mai mici decât altele petrecute în istorie, fără a lua în calcul și preistoria.
În ce privește schimbarile climatice globale influențate de activitatea umană sunt foarte sceptic, nu a reușit  nimeni să-mi vină cu argumente științifice prin care să-mi demonstreze că activitatea umană ar avea o influență, oricât de mică dar detectabilă, asupra climei la nivel global.
Deasemenea nu sunt de acord nu felul în care sunt studiate schimbările climatice sau cel puțin nu sunt de acord cu modul în care sunt prezentate publicului aceste studii.
Sunt convins că modelele prin care se fac prognoze pe termen relativ lung 10 ani, 1000 ani sunt irelevante în stadiul actual de dezvoltare a tehnologiei. Chestii de genul temperatura va crește/scade cu  grade în următorii  ani unde  mi se par fabulații lipsite de orice bază științifică.

Acuma ca fapt divers -- cu o mare parte din poziţia asta sunt şi eu de acord. Schimbarea actuală nu e cu nimic mai mare decât vârful de încălzire medieval, nici decât răcirea din secolele 14-18, care n-au avut nici pe departe efecte catastrofale. Corelaţia nu înseamnă automat implicaţie, deci magnitudinea rolului jucat de activitatea antropică e greu de cuantificat, şi dat fiind stadiul actual al modelelor climatice, predicţii pe mai mult de 1 an sunt greu de făcut. Hell, ăştia nu nimeresc de fiecare dată starea vremii peste 3 zile.
Păi nu e bine, dacă ajungem să fim de acord o să ne dăm dreptate și nu mai acumulăm nici unul din noi informație și nici cei care citesc discuția, preferam să mă contrazici.
Ai spus că ești de acord doar ,,cu o mare parte din poziţia asta", cu ce nu ești de acord? Poate iese ceva de aici.
Eu am spus ,,nu a reușit  nimeni să-mi vină cu argumente științifice prin care să-mi demonstreze că activitatea umană ar avea o influență, oricât de mică dar detectabilă, asupra climei la nivel global" iar tu ai spus ,,magnitudinea rolului jucat de activitatea antropică e greu de cuantificat", cunoști vreo metodă, chiar și ,,grea", prin care la nivelul teoretic și tehnologic actual am putea să ,,cuantificăm magnitudinea rolului jucat de activitatea antropică"?
Titlu: Re: Măsurarea temperaturii atmosferice și punerea în evidență a schimbărilor climatice
Scris de: Mihnea Maftei din Februarie 02, 2011, 06:35:13 PM
Inclusiv erorile sistematice (la nivel de statie meteo) se diminueaza prin mediere, cand vorbim de multe statii meteo. Unele statii vor avea erorile sistematice pozitive, altele negative etc. E greu de crezut ca acum 100 de ani mai toate erorile sistematice erau negative, iar acum mai toate erorile sistematice sunt pozitive, astfel incat sa rezulte in mod fals ca temperatura a crescut.
Titlu: Re: Măsurarea temperaturii atmosferice și punerea în evidență a schimbărilor climatice
Scris de: AlexandruLazar din Februarie 02, 2011, 06:40:08 PM
CitatLanțul tău de măsură este format dintr-un senzor care transformă temperatura într-o marime electrică, tensiune, curent, rezistență [/b]nu are importanță[/b].
Mai apoi mărimea electrică o aplici unui convertor A/D, în general înainte de convertor ai un circuit care reține valoarea mărimii electrice pe parcursul măsurării, avea un nume dar îmi scapă acum pentru că nu am mai lucrat de mult. Convertorul A/D poate fi cu integrare în timp (poate greșesc termeni dar înțelegi tu) sau poate avea un comparator un convertor D/A și un registru cu aproximații succesive, nu cred că aici se aplică alte variante decât astea ținând cont că vrem cel puțin 10 bit. Undeva în lanț, cât timp avem mărimea electrică vor apare filtre pentru eliminarea zgomotelor.
Acum gândeștete unde în lanțul acesta apare medierea și ne putem da seama ce mărime se mediază.

Din păcate are importanţă pentru că CAD nu ştie să convertească decât tensiuni :). Ca să discutăm la obiect, de fapt pentru astfel de măsurători se foloseşte de regulă un montaj cu sursă de curent constant care alimentează direct senzorul (şi se culege căderea de tensiune de pe el). Mai rar merge varianta cu punte Wheatsone în regim neechilibrat pentru că sensibilitatea unui senzor de temperatură de precizie e oricum foarte mică, dacă mai trebuie s-o împarţi şi la patru s-a dus treaba.

CAD poate fi într-adevăr cu integrare în timp (dublă rampă, triplă sau câtă o fi) sau cu aproximaţii succesive. Acuma, întrucât temperatura este o mărime foarte lent variabilă, de regulă nu se foloseşte şi un circuit de eşantionare şi memorare. Unde apare medierea:

a) Dacă CAN este un CAN cu integrare în timp, caracteristica lui de conversie este de tipul [tex]U_{out}=U_{ref} \cdot \frac{N_1}{N_{down}}, unde [tex]N_1[/tex] este numărul de perioade ale ceasului în timpul de integrare, [tex]T_i[/tex] pentru tensiunea [tex]V_{in}[/tex] iar [tex]N_{down}[/tex] este numărul de perioade ale ceasului în care, pornind de la tensiunea la care a ajuns integratorul, se scade tensiunea de a ieşire până la 0. Constanta de timp a integratorului, [tex]RC[/tex] se ia egală cu timpul de integrare, astfel încât ceea ce ai după timpul de integrare [tex]T_{i}[/tex] la ieşirea integratorului este [tex]\frac{1}{T_{i}}\int_{0}^{T_i} U(\theta) dt[/tex] unde tensiunea aplicată depinde proporţional de [tex]\theta[/tex] şi îi urmăreşte variaţiile (cu un oareşice decalaj pentru că timpul de răspuns nu e instantaneu, dar orişicât). În final ai o relaţie de tipul [tex]k\cdot\frac{1}{T_{i}}\int_{0}^{T_i} \theta(t) dt[/tex]. Oricum, morala e că tensiunea de ieşire din integrator, şi implicit valoarea convertită, nu depinde de tensiunea scoasă de senzor la începutul integrării (pentru că nu e perfect constantă pe timpul integrării, iar integrarea nu se face instantaneu!) ci de valoarea ei pe timpul integrării. Tensiunea variază odată cu rezistenţa senzorului, e drept că nu perfect sincron cu temperatura din cauza timpului de răspuns limitat, dar nu e permanent constantă, aşa că ieşirea lantului nu depinde numai de o temperatură, la un singur moment, ci de temperaturile de la diverse momente de timp.

Edit: o văd venind aşa că răspund de acuma ;D. În cărţi o să găseşti raportul nu între număr de pulsuri ci între durate (t_up/t_down). Evident însă că în CAN nu e niciun spiriduş cu cronometru care împarte la sfârşit rezultatele, de fapt cronometrarea se face prin numărarea unor impulsuri de ceas. De asemenea, convertorul cu o singură pantă e mai mult un model ideal, de fapt nu se foloseşte în montaje reale.

b) Dacă e vorba de un CAN cu aproximaţii succesive e un pic mai greu de scos explicit relaţia de mediere din cauza comparatorului, dar putem face chestia asta intuitiv: caracteristica lui de conversie este de tipul [tex]V_{out}=V_{ref}\cdot\frac{B_{1}2^{n-1}+...+B_{n}2^0}{2^n}[/tex], unde biţii B1..n sunt obţinuţi prin comparaţie între tensiunea de referinţă pentru bitul de fiecare ordin şi tensiunea [tex]V_{in}[/tex]. Întrucât conversia asta, din nou, nu se face instantaneu, tensiunea de intrare [tex]V_{in}[/tex] comparată cu [tex]V_{ref}[/tex] depinde de fiecare dată de temperatura de la un alt moment de timp: dacă fiecare bit se obţine într-un timp [tex]t_c[/tex], bitul 1 depinde de temperatura la [tex]t_c[/tex], bitul 2 depinde de temperatura la [tex]2t_c[/tex] şi tot aşa. Deci valoarea pe care o scoate la ieşire este, oricum dacă nu o medie, cel puţin o combinaţie liniară care depinde de temperaturile de la momente de timp diferite.

Asta nu afectează cu nimic "sensul" mărimii de ieşire: pe hârtie n-are sens, dar timpii de care vorbim sunt atât de mici încât, dacă nu te supără neortodoxia faţă de caracterul intensiv al mărimilor, obţii ceva cât se poate de valid :).

Nu se mediază direct temperatura, de acord, dar caracteristica de conversie (în care apare o mediere a temperaturii, explicită în primul caz, sau oricum o combinaţie liniară de temperaturi de la momente de timp diferite în al doilea) este în raport cu temperatura de intrare, nu cu tensiunea de undeva de la jumătatea lanţului de măsurare. Dependenţa între mărimile din lanţ si temperatură este una continuă în timp pentru că un lanţ cu componente digitale nu va funcţiona niciodată instantaneu.

CitatPăi nu e bine, dacă ajungem să fim de acord o să ne dăm dreptate și nu mai acumulăm nici unul din noi informație și nici cei care citesc discuția, preferam să mă contrazici.
Ai spus că ești de acord doar ,,cu o mare parte din poziţia asta", cu ce nu ești de acord? Poate iese ceva de aici.

Cu asta nu sunt de acord:

CitatÎn mod subiectiv, de data aceasta, consider că schimbările climatice, din această perioadă istorică, nu sunt deloc spactaculoase și sunt oricum mult mai mici decât altele petrecute în istorie, fără a lua în calcul și preistoria.

Nu e vorba de nişte schimbări mult mai mici ci de unele comparabile cu altele din istorie, de exemplu cu încălzirea pronunţată din timpul Evului Mediu sau cu răcirea accentuată care i-a urmat. Acestea nu au avut niciodată urmări catastrofale însă au avut influenţe importante asupra dezvoltării umane. Şirul lung de ierni geroase şi toamne cu precipitaţii abundente si temperaturi scăzute din timpul secolelor 17 şi 18 au afectat serios recoltele -- în asemenea măsură încât în ţările cele mai afectate (de exemplu Irlanda) oamenii se alegeau cu câte o foamete la fiecare 10-12 ani. Randamentul slab al recoltelor din Europa şi imposibilitatea de aclimatizare a unor plante a încurajat dezvoltarea imperiilor coloniale şi activităţile agricole de acolo. Similar, condiţiile mult mai blânde din timpul Evului Mediu au favorizat creşterea demografică spectaculoasă din acea perioadă.

CitatEu am spus ,,nu a reușit  nimeni să-mi vină cu argumente științifice prin care să-mi demonstreze că activitatea umană ar avea o influență, oricât de mică dar detectabilă, asupra climei la nivel global" iar tu ai spus ,,magnitudinea rolului jucat de activitatea antropică e greu de cuantificat", cunoști vreo metodă, chiar și ,,grea", prin care la nivelul teoretic și tehnologic actual am putea să ,,cuantificăm magnitudinea rolului jucat de activitatea antropică"?

Niciuna realizabilă :). Din păcate nu avem o copie exactă a Pământului în care să avem aceleaşi condiţii naturale dar să nu avem activitate antropică. De-asta spun că este greu de cuantificat: în lipsa unei referinţe, putem cel mult să ne dăm cu presupusul pe baza efectelor pe care presupunem că le-ar avea diversele gaze cu efect de seră. Cunoaştem impactului efectului de seră la scară planetară de pe planete ca Venus dar până acolo e cale lungă.

Nu-i bai, şi mie îmi place contrazicerea  ;D N-am mai stat atâta pe forumul ŞtiinţaAzi de nu ştiu când, şi cred că-i cam prima dată când o fac constructiv, până acuma dezbateri d-astea aveam numai cu creaţionişti sau cu unii cu fantezii şi mai grozave.
Titlu: Re: Măsurarea temperaturii atmosferice și punerea în evidență a schimbărilor climatice
Scris de: mircea_p din Februarie 02, 2011, 07:53:21 PM
Citat din: tavy din Februarie 02, 2011, 09:56:40 AM
Legat de 1.: Erorile de măsură dispar în foarte mică măsură prin mediere.
Să facem un calcul:
Dacă avem masurătorile:
[tex]T_1, T_2, \dots, T_n[/tex]
Masurate cu erorile absolute:
[tex]\delta T_1, \delta T_2 \dots, \delta T_n[/tex]
Media va fi:
[tex]\overline{T} = \frac{\sum_{i=1}^{n} T_i} {n}[/tex]
Iar eroarea absolută va fi:
[tex]\delta\overline{T} = \frac{\sum_{i=1}^{n} \delta T_i} {n}[/tex]
Dacă presupunem că toate masurătorile au fost făcute cu aceeași eroare relativă [tex]\delta T[/tex] atunci eroarea absolută a mediei va fi tot [tex]\delta T[/tex], poți observa că prin mediere nu am obținut o imbunătățire a erorii de măsură.
Eroarea delta T nu este relativa ci absoluta. Eroarea relativa este deltaT/T. Nu ca ar avea imposrtanata pentru argument.

Acum, ceea ce spui ar fi adevarat daca delta T ar fi o constanta. Asa ar putea fi adevarat pentru o eroare sistematica, cum a mentionat si Alex Lazar.
Daca insa eroarea este un parametru aleator, atunci  insa media este fie zero  (in cazul unei distributii simetrice) sau o valoarea oarecare (diferita de maximul erorii) in cazul unei distributii nesimetrice. Asta este statistica de baza.
Cand spunem ca instrumentul masoarea cu (+/-) 0.5 grade insemna ca eroarea poate fi oriunde intre -0.5 si +0.5, nu ca este tot timpul +0.5 (daca ar fi fixa si cunoscuta, am putea sa o scadem pur si simplu, ce rost ar mai avea sa ne batem capul cu ea?).
Media erorilor facute in diferite masuratori cu acest instrument nu este este 0.5 ci (ideal) zero.
Sigur ca in practica te poti intreba daca eroarea are distribuite gausiana sau nu, daca e simetrica sau nu, dar in principiu medierea (poate) reduce erorile aleatoare.

Titlu: Re: Măsurarea temperaturii atmosferice și punerea în evidență a schimbărilor climatice
Scris de: mircea_p din Februarie 02, 2011, 08:16:00 PM
Citat din: tavy din Februarie 02, 2011, 09:56:40 AM
Citat din: mircea_p din Februarie 02, 2011, 07:15:54 AM
Mi se pare ca problema aici este care este semnificatia modificarilor in temperatura medie masurate de climatologi si daca se pot sau nu trage concluzii in privinta evolutiei viitoare.
Chiar daca ceea ce masoara ei nu apare in vreo formula de fizica, nu inseamna ca nu exista. Cel putin nu in mod necesar.
Chiar dacă nu putem detecta inorogul roz indetectabil care trăiește în centrul pământului, nu înseamnă că nu există.
Nu vad cum se aplica inorogul aici. Nu discutam existenta sau non-existenta unei entitati aici. Sau eu nu am inteles asta.
Era vorba de sensul sau nonsensul unor masuratori si/sau operatii matematice. Ori vrei sa spui ca diferite medii ale temepraturii nici macar nu exista? Nu cred ca e cazul sa deviem acum in discutii pe tema "ce inseamna ca ceva exista"?
Se poate calcula media oricaror numere. Din punct de vedere matematic are sens. Si exista.
Dar discutia nu era in nici un caz despre asa ceva.

Citat din: tavy din Februarie 02, 2011, 09:56:40 AM
Citat din: mircea_p din Februarie 02, 2011, 07:15:54 AM
Obstinatia asta cu fizica si parametrii intensivi aduce cumva cu faimoasa argumentare de tipul "om de paie" (atribui altora afirmatii pe care nu le-au facut si apoi le combati) . A zis cineva ca mediile de temepratura folosite in meteorologie sant acelasi lucru cu temperatura termodinamica?
Nu trebuie să fie același lucru cu temperatura termodinamică, trebuie doar să aibă o semnificație fizică pentru că altfel nu poate fi folosită decât în numerologie climatologie.
Acum chiar ca nu mai inteleg.
Poti sa definesti ce inseamna "semnificatie fizica", ca sa fie clar?
Pot exista si alte semnificatii, diferite de "semnificatie fizica"?

As fi adugat si eu ca sant de acord cu observatiile tale in privinta climatice. Dar inteleg ca nu am voie sa fiu de acord cu tine :)
Totusi ce s-a observat (si nu numai de mine) este ca de cand politica a devenit un factor, ambele "tabere" s-au facut vinovate de distorsiuni partizane. De asemena, la fel de clar, distorsiunile, erorile si interpretarea gresita scad credibilitatea ambelor grupuri.





Titlu: Re: Măsurarea temperaturii atmosferice și punerea în evidență a schimbărilor climatice
Scris de: tavy din Februarie 02, 2011, 08:31:17 PM
Citat din: AlexandruLazar din Februarie 02, 2011, 06:40:08 PM
CitatLanțul tău de măsură este format dintr-un senzor care transformă temperatura într-o mărime electrică, tensiune, curent, rezistență [/b]nu are importanță[/b].
Din păcate are importanţă pentru că CAD nu ştie să convertească decât tensiuni :).
Când am spus că nu are importanță mă gândeam că oricum se pot face foarte ușor transformări între cele trei mărimi.
Dacă stau mai bine și mă gândesc convertorul cu integrare convertește de fapt curentul (integrarea în timp a curentului) și nu tensiunea, rezistența care o punem înainte de punctul de nul virtual (parcă așa îi zicea, a trecut ceva vreme ce când am lucrat cu A.O.) este un artificiu prin care transformăm tensiunea de intrare în curent. În fapt, circuitul integrator, poate fi văzut ca un circuit care încarcă un condensator cu curentul de intrare.

Citat din: AlexandruLazar din Februarie 02, 2011, 06:40:08 PM
Ca să discutăm la obiect, de fapt pentru astfel de măsurători se foloseşte de regulă un montaj cu sursă de curent constant care alimentează direct senzorul (şi se culege căderea de tensiune de pe el). Mai rar merge varianta cu punte Wheatsone în regim neechilibrat pentru că sensibilitatea unui senzor de temperatură de precizie e oricum foarte mică, dacă mai trebuie s-o împarţi şi la patru s-a dus treaba.
La fel de bine, teoretic, putem folosi o tensiune constantă iar senzorul să ia locul rezistenței din integrator. Nu am făcut calcule dar nu te grăbi să-mi zici că nu merge înainte să te gândești bine că am mai ai văzut un conferențiar acum 18 ani care susținea că nu am cum cu un DAC08, un comparator și un calculator (COBRA) să fac un convertor A/D fară să folosesc un registru de aproximații succesive sau un numărător UP/DOWN și pe urmă nu stia ce să mai zică când l-a văzut funcționând. Bineînțeles schema mea mi-a fost predată în anul patru de același conferențiar la cursul ,,tehnici moderne de măsură".
Citat din: AlexandruLazar din Februarie 02, 2011, 06:40:08 PM
CAD poate fi într-adevăr cu integrare în timp (dublă rampă, triplă sau câtă o fi) sau cu aproximaţii succesive. Acuma, întrucât temperatura este o mărime foarte lent variabilă, de regulă nu se foloseşte şi un circuit de eşantionare şi memorare. Unde apare medierea:

a) Dacă CAN este un CAN cu integrare în timp, caracteristica lui de conversie este de tipul [tex]U_{out}=U_{ref} \cdot \frac{N_1}{N_{down}}, unde [tex]N_1[/tex] este numărul de perioade ale ceasului în timpul de integrare, [tex]T_i[/tex] pentru tensiunea [tex]V_{in}[/tex] iar [tex]N_{down}[/tex] este numărul de perioade ale ceasului în care, pornind de la tensiunea la care a ajuns integratorul, se scade tensiunea de a ieşire până la 0. Constanta de timp a integratorului, [tex]RC[/tex] se ia egală cu timpul de integrare, astfel încât ceea ce ai după timpul de integrare [tex]T_{i}[/tex] la ieşirea integratorului este [tex]\frac{1}{T_{i}}\int_{0}^{T_i} U(\theta) dt[/tex] unde tensiunea aplicată depinde proporţional de [tex]\theta[/tex] şi îi urmăreşte variaţiile (cu un oareşice decalaj pentru că timpul de răspuns nu e instantaneu, dar orişicât). În final ai o relaţie de tipul [tex]k\cdot\frac{1}{T_{i}}\int_{0}^{T_i} \theta(t) dt[/tex]. Oricum, morala e că tensiunea de ieşire din integrator, şi implicit valoarea convertită, nu depinde de tensiunea scoasă de senzor la începutul integrării (pentru că nu e perfect constantă pe timpul integrării, iar integrarea nu se face instantaneu!) ci de valoarea ei pe timpul integrării. Tensiunea variază odată cu rezistenţa senzorului, e drept că nu perfect sincron cu temperatura din cauza timpului de răspuns limitat, dar nu e permanent constantă, aşa că ieşirea lantului nu depinde numai de o temperatură, la un singur moment, ci de temperaturile de la diverse momente de timp.

Nu se mediază direct temperatura, de acord, dar caracteristica de conversie (în care apare o mediere a temperaturii, explicită în primul caz, sau oricum o combinaţie liniară de temperaturi de la momente de timp diferite în al doilea) este în raport cu temperatura de intrare, nu cu tensiunea de undeva de la jumătatea lanţului de măsurare. Dependenţa între mărimile din lanţ si temperatură este una continuă în timp pentru că un lanţ cu componente digitale nu va funcţiona niciodată instantaneu.
Deci recunoști că nu se mediază temperatura, de altfel suntem obligați din definiția temperaturii să considerăm că în timpul măsurătorii aceasta nu se modifică, temperatura nu este definită decât pentru sisteme la echilibru termodinamic.
La [tex]k\cdot\frac{1}{T_{i}}\int_{0}^{T_i} \theta(t) dt[/tex] ai ajuns dând niște factori comuni nu? În realitate în integrală nu-ți intra doar temperatura ca funcție de timp.

Citat din: AlexandruLazar din Februarie 02, 2011, 06:40:08 PM
o văd venind aşa că răspund de acuma ;D. În cărţi o să găseşti raportul nu între număr de pulsuri ci între durate (t_up/t_down). Evident însă că în CAN nu e niciun spiriduş cu cronometru care împarte la sfârşit rezultatele, de fapt cronometrarea se face prin numărarea unor impulsuri de ceas. De asemenea, convertorul cu o singură pantă e mai mult un model ideal, de fapt nu se foloseşte în montaje reale.

Ți-ai format o impresie gresită despre mine, lucrarea me de diplomă s-a numit ,,Sistem cu microprocesor Z80 pentru automatizarea operațiilor de masură". Era vorba de o placă de bază cu microprocesor Z80 cu interfață IEEE-488 și cu niște conectori care emula ISA pe 8 bit, pentru acest sistem am proiectat bineînțeles și un convertor A/D.
Să nu râzi de mine cu proiectul acesta, gândeștete că era mijlocul anilor '90 și nu eram student la automatică.

Citat din: AlexandruLazar din Februarie 02, 2011, 06:40:08 PM
Cu asta nu sunt de acord:
CitatÎn mod subiectiv, de data aceasta, consider că schimbările climatice, din această perioadă istorică, nu sunt deloc spactaculoase și sunt oricum mult mai mici decât altele petrecute în istorie, fără a lua în calcul și preistoria.
Nu e vorba de nişte schimbări mult mai mici ci de unele comparabile cu altele din istorie, de exemplu cu încălzirea pronunţată din timpul Evului Mediu sau cu răcirea accentuată care i-a urmat. Acestea nu au avut niciodată urmări catastrofale însă au avut influenţe importante asupra dezvoltării umane. Şirul lung de ierni geroase şi toamne cu precipitaţii abundente si temperaturi scăzute din timpul secolelor 17 şi 18 au afectat serios recoltele -- în asemenea măsură încât în ţările cele mai afectate (de exemplu Irlanda) oamenii se alegeau cu câte o foamete la fiecare 10-12 ani. Randamentul slab al recoltelor din Europa şi imposibilitatea de aclimatizare a unor plante a încurajat dezvoltarea imperiilor coloniale şi activităţile agricole de acolo. Similar, condiţiile mult mai blânde din timpul Evului Mediu au favorizat creşterea demografică spectaculoasă din acea perioadă.
Din păcate aici nu pot să te contrazic, nu ma pricep la chestia asta și din cauza asta am și spus că sunt subiectiv.

Citat din: AlexandruLazar din Februarie 02, 2011, 06:40:08 PM
CitatEu am spus ,,nu a reușit  nimeni să-mi vină cu argumente științifice prin care să-mi demonstreze că activitatea umană ar avea o influență, oricât de mică dar detectabilă, asupra climei la nivel global" iar tu ai spus ,,magnitudinea rolului jucat de activitatea antropică e greu de cuantificat", cunoști vreo metodă, chiar și ,,grea", prin care la nivelul teoretic și tehnologic actual am putea să ,,cuantificăm magnitudinea rolului jucat de activitatea antropică"?

Niciuna realizabilă :). Din păcate nu avem o copie exactă a Pământului în care să avem aceleaşi condiţii naturale dar să nu avem activitate antropică. De-asta spun că este greu de cuantificat: în lipsa unei referinţe, putem cel mult să ne dăm cu presupusul pe baza efectelor pe care presupunem că le-ar avea diversele gaze cu efect de seră. Cunoaştem impactului efectului de seră la scară planetară de pe planete ca Venus dar până acolo e cale lungă.

Doar cele realizabile sunt relevante.

În ce privește efectul de seră de pe Venus te rog să-mi permiți să fiu sceptic, mai ales că eu știu ce înseamnă efect de seră.
Pentru cei care nu știu, efectul de seră are loc atunci când radiația pătrunde în sistem iar sistemul poate pierde puțină căldură radiativ. Pentru ca efectul de seră să poată fi efectiv trebuie redusă și pierderea de energie prin convecție. Atmosfera, atât pe Pământ cât și pe Venus nu respectă partea cu convecția în sensul că energia poate ajunge din straturile inferioare ale atmosferei în cele superioare foarte ușor prin convecție. Odată energia ajunsă în straturile superioare ale atmosferei se poate pierde nestingherită de cele 400ppm dioxid de carbon din atmosfera pământului. (400ppm e scos cam din burtă dar cred că e cam pe acolo).
Pe de altă parte, admițând că CO2 ar contribui la efectul de seră pe Venus și pe Pământ, atmosfera pe Venus are 96% CO2 iar pe Pământ 0,0x% CO2.
Poate distanța între Venus și Soare să aibă o importanță mai mare în faptul că temperatura pe venus este mai mare decât pe Pământ?
Titlu: Re: Măsurarea temperaturii atmosferice și punerea în evidență a schimbărilor climatice
Scris de: tavy din Februarie 02, 2011, 08:45:10 PM
Citat din: mircea_p din Februarie 02, 2011, 07:53:21 PM
Citat din: tavy din Februarie 02, 2011, 09:56:40 AM
Legat de 1.: Erorile de măsură dispar în foarte mică măsură prin mediere.
Să facem un calcul:
Dacă avem masurătorile:
[tex]T_1, T_2, \dots, T_n[/tex]
Masurate cu erorile absolute:
[tex]\delta T_1, \delta T_2 \dots, \delta T_n[/tex]
Media va fi:
[tex]\overline{T} = \frac{\sum_{i=1}^{n} T_i} {n}[/tex]
Iar eroarea absolută va fi:
[tex]\delta\overline{T} = \frac{\sum_{i=1}^{n} \delta T_i} {n}[/tex]
Dacă presupunem că toate masurătorile au fost făcute cu aceeași eroare relativă [tex]\delta T[/tex] atunci eroarea absolută a mediei va fi tot [tex]\delta T[/tex], poți observa că prin mediere nu am obținut o imbunătățire a erorii de măsură.
Eroarea delta T nu este relativa ci absoluta. Eroarea relativa este deltaT/T. Nu ca ar avea imposrtanata pentru argument.
Evident [tex]\delta T[/tex] este eroare absolută, a fost o greșală de redactare, cred că reiese asta pentru că am definit mai devreme ce ințeleg prin [tex]\delta[/tex].

Citat din: tavy din Februarie 02, 2011, 09:56:40 AM
Acum, ceea ce spui ar fi adevarat daca delta T ar fi o constanta. Asa ar putea fi adevarat pentru o eroare sistematica, cum a mentionat si Alex Lazar.
Daca insa eroarea este un parametru aleator, atunci  insa media este fie zero  (in cazul unei distributii simetrice) sau o valoarea oarecare (diferita de maximul erorii) in cazul unei distributii nesimetrice. Asta este statistica de baza.
Cand spunem ca instrumentul masoarea cu (+/-) 0.5 grade insemna ca eroarea poate fi oriunde intre -0.5 si +0.5, nu ca este tot timpul +0.5 (daca ar fi fixa si cunoscuta, am putea sa o scadem pur si simplu, ce rost ar mai avea sa ne batem capul cu ea?).
Media erorilor facute in diferite masuratori cu acest instrument nu este este 0.5 ci (ideal) zero.
Sigur ca in practica te poti intreba daca eroarea are distribuite gausiana sau nu, daca e simetrica sau nu, dar in principiu medierea (poate) reduce erorile aleatoare.
[tex]\delta T[/tex] este eroarea maximă dată de măsurătoare, bineînțeles că nu cunoaștem eroarea reală pentru că asta ar presupune că cunoaștem valoarea exactă a mărimii și atunci nu mai are rost măsurătoarea.

Citat din: mircea_p din Februarie 02, 2011, 08:16:00 PM
Citat din: tavy din Februarie 02, 2011, 09:56:40 AM
Chiar dacă nu putem detecta inorogul roz indetectabil care trăiește în centrul pământului, nu înseamnă că nu există.
Nu vad cum se aplica inorogul aici. Nu discutam existenta sau non-existenta unei entitati aici. Sau eu nu am inteles asta.
Discutăm despre relevanța existenței aici.

Citat din: mircea_p din Februarie 02, 2011, 08:16:00 PM
Era vorba de sensul sau nonsensul unor masuratori si/sau operatii matematice. Ori vrei sa spui ca diferite medii ale temepraturii nici macar nu exista? Nu cred ca e cazul sa deviem acum in discutii pe tema "ce inseamna ca ceva exista"?
Se poate calcula media oricaror numere. Din punct de vedere matematic are sens. Si exista.
Dar discutia nu era in nici un caz despre asa ceva.
Ok, putem calcula media oricăror numere, și media numerelor de telefon din agendă. Nu înseamnă că respectiva medie are vre-o semnificație.
Titlu: Re: Măsurarea temperaturii atmosferice și punerea în evidență a schimbărilor climatice
Scris de: mircea_p din Februarie 02, 2011, 09:36:47 PM
Citat din: tavy din Februarie 02, 2011, 08:45:10 PM
[tex]\delta T[/tex] este eroarea maximă dată de măsurătoare, bineînțeles că nu cunoaștem eroarea reală pentru că asta ar presupune că cunoaștem valoarea exactă a mărimii și atunci nu mai are rost măsurătoarea.
Ok, nu inteleg ce vrei sa spui cu asta. Faptul ca nu stim valoarea reala este irelevanta pentru analiza statistica.
Tot mai crezi ca media erorilor aleatoare este egala cu eroarea unei singure masuratori?
Titlu: Re: Măsurarea temperaturii atmosferice și punerea în evidență a schimbărilor climatice
Scris de: AlexandruLazar din Februarie 02, 2011, 10:17:37 PM
CitatCând am spus că nu are importanță mă gândeam că oricum se pot face foarte ușor transformări între cele trei mărimi.
Dacă stau mai bine și mă gândesc convertorul cu integrare convertește de fapt curentul (integrarea în timp a curentului) și nu tensiunea, rezistența care o punem înainte de punctul de nul virtual (parcă așa îi zicea, a trecut ceva vreme ce când am lucrat cu A.O.) este un artificiu prin care transformăm tensiunea de intrare în curent. În fapt, circuitul integrator, poate fi văzut ca un circuit care încarcă un condensator cu curentul de intrare.

Depinde din ce direcţie te uiţi ;D. Pentru un convertor dublă-pantă se pot privi lucrurile şi aşa, însă mărimea primară de intrare care variază e totuşi căderea de tensiune pe senzor. Pentru alte tipuri de convertor (de exemplu pentru convertoare sigma-delta, care sunt mai uşor de folosit pe sisteme integrate) nu mai e valabilă asta.

CitatLa fel de bine, teoretic, putem folosi o tensiune constantă iar senzorul să ia locul rezistenței din integrator. Nu am făcut calcule dar nu te grăbi să-mi zici că nu merge înainte să te gândești bine că am mai ai văzut un conferențiar acum 18 ani care susținea că nu am cum cu un DAC08, un comparator și un calculator (COBRA) să fac un convertor A/D fară să folosesc un registru de aproximații succesive sau un numărător UP/DOWN și pe urmă nu stia ce să mai zică când l-a văzut funcționând. Bineînțeles schema mea mi-a fost predată în anul patru de același conferențiar la cursul ,,tehnici moderne de măsură".

Se poate desigur şi aşa, toată ideea e să obţii o pantă proporţională cu tensiunea de intrare. Soluţia asta nu e foarte convenabilă la scară industrială dar evident că e perfect posibil :).


CitatDeci recunoști că nu se mediază temperatura, de altfel suntem obligați din definiția temperaturii să considerăm că în timpul măsurătorii aceasta nu se modifică, temperatura nu este definită decât pentru sisteme la echilibru termodinamic.
La [tex]k\cdot\frac{1}{T_{i}}\int_{0}^{T_i} \theta(t) dt[/tex] ai ajuns dând niște factori comuni nu? În realitate în integrală nu-ți intra doar temperatura ca funcție de timp.

Da, desigur. Sub integrală e de fapt o tensiune care depinde de temperatură; dependenţa e prin variaţia rezistenţei în raport cu temperatura, dar dacă scoţi în faţă toate constantele rămâne sub integrală doar dependenţa de temperatura -- celelalte le-am lăsat în faţă sub forma k-ului de acolo. Sigur că nu se mediază explicit temperatura, în fond în CAN nu ai nicio temperatură. Dar rezultatul este o tensiune care depinde de temperaturi de la momente diferite. Adică, tensiunea de intrare pe care se apucă convertorul dublă-pantă să o integreze (şi care depinde de temperatură) nu e aceeaşi atunci când începe ca atunci când termină. Când, din ceea ce a scos CAN-ul, se face reversarea modelului ca să ajungi de la numărul scos de CAN la temperatura de la care a pornit toată treaba, rezultatul e o singură tensiune, fixă, de la care se ajunge la o singură temperatură, cu toate că în realitate CAN-ul a lucrat cu o tensiune care n-a fost riguros constantă pentru că temperatura de care ea depindea n-ar fi avut cum să fie riguros constantă.

Acu' bineînţeles că un elementar bun-simţ ingineresc spune (pe bună dreptate ;D) că n-are importanţă, dar riguros ar trebui să n-aibă sens ce iese acolo. Similar, sigur că din datele climatologice, adunate şi împărţite în fel şi chip, s-ar putea extrapole multe prostii. Asta nu înseamnă că tot ce iese din ele e incorect.

CitatȚi-ai format o impresie gresită despre mine, lucrarea me de diplomă s-a numit ,,Sistem cu microprocesor Z80 pentru automatizarea operațiilor de masură". Era vorba de o placă de bază cu microprocesor Z80 cu interfață IEEE-488 și cu niște conectori care emula ISA pe 8 bit, pentru acest sistem am proiectat bineînțeles și un convertor A/D.
Să nu râzi de mine cu proiectul acesta, gândeștete că era mijlocul anilor '90 și nu eram student la automatică.

Tocmai pentru că ştiu ce înseamnă asta nici nu am de gând să râd :).

CitatPoate distanța între Venus și Soare să aibă o importanță mai mare în faptul că temperatura pe venus este mai mare decât pe Pământ?

Nu numai asta, ci şi perioada de rotaţie foarte scăzută care înseamnă că ciclul de încălziri şi răciri a fiecărui punct de pe suprafaţă se realizează mult mai lent.
Titlu: Re: Măsurarea temperaturii atmosferice și punerea în evidență a schimbărilor climatice
Scris de: mircea_p din Februarie 02, 2011, 10:27:04 PM
Citat din: tavy din Februarie 02, 2011, 08:31:17 PM
Poate distanța între Venus și Soare să aibă o importanță mai mare în faptul că temperatura pe venus este mai mare decât pe Pământ?
Chiar este? Cum putem sti daca temperatura pe Venus e mai mare? Ce sens fizic are termenul "temperatura pe Venus"? Ori presupui ca intreaga planeta e la echilibru termic?
Titlu: Re: Măsurarea temperaturii atmosferice și punerea în evidență a schimbărilor climatice
Scris de: mircea_p din Februarie 02, 2011, 10:38:19 PM
In legatura cu pierderile prin convectie, un site NASA arata o imagine cu importanta diferitelor mecanisme de pierdere de caldura.
Dupa ei, 5% se pierde prin convectie si 17% prin radiatie.
Eu nu am mijloacele de a verifica asta, nici experimental nici teoretic, din pacate.
Titlu: Re: Măsurarea temperaturii atmosferice și punerea în evidență a schimbărilor climatice
Scris de: Adi din Februarie 02, 2011, 10:41:08 PM
Citat din: mircea_p din Februarie 02, 2011, 10:27:04 PM
Citat din: tavy din Februarie 02, 2011, 08:31:17 PM
Poate distanța între Venus și Soare să aibă o importanță mai mare în faptul că temperatura pe venus este mai mare decât pe Pământ?
Chiar este? Cum putem sti daca temperatura pe Venus e mai mare? Ce sens fizic are termenul "temperatura pe Venus"? Ori presupui ca intreaga planeta e la echilibru termic?


Haha, l-ai prins aici!
Titlu: Re: Măsurarea temperaturii atmosferice și punerea în evidență a schimbărilor climatice
Scris de: tavy din Februarie 02, 2011, 11:01:24 PM
Citat din: mircea_p din Februarie 02, 2011, 09:36:47 PM
Citat din: tavy din Februarie 02, 2011, 08:45:10 PM
[tex]\delta T[/tex] este eroarea maximă dată de măsurătoare, bineînțeles că nu cunoaștem eroarea reală pentru că asta ar presupune că cunoaștem valoarea exactă a mărimii și atunci nu mai are rost măsurătoarea.
Ok, nu inteleg ce vrei sa spui cu asta. Faptul ca nu stim valoarea reala este irelevanta pentru analiza statistica.
Tot mai crezi ca media erorilor aleatoare este egala cu eroarea unei singure masuratori?
O să încerc să fiu mai explicit.
Presupunem că masurăm mărimea fizică [tex]M[/tex] cu un aparat de măsură și obținem valoarea [tex]v[/tex]. În acest moment știm că valoarea mărimii [tex]M[/tex], [tex]v_{real}[/tex] se găsește în intervalul [tex]\left [ v-\delta v, v+\delta v \right ][/tex], unde [tex]\delta v[/tex] este dat de modul în care fac măsurătoarea, acest [tex]\delta v[/tex] îl numesc eu eroare în contextul dat.
Acum, noi știm că [tex]v_{real}[/tex] se găsește în intervalul [tex]\left [ v-\delta v, v+\delta v \right ][/tex] cu o anumită certitudine (probabilitate), cu alte cuvinte există o anumită încertitudine, incertitudine datorată factorilor aleatori și care poate fi estimată cu oarece precizie dar modul de estimare deja este alt subiect și prea complicat ca să-l explic aici (ne facem că nu observăm că am uitat cum se face).
Mergem mai departe și facem [tex]N[/tex] masurători ale mărimii fizice [tex]M[/tex] obținând valorile [tex]v_1, v_2, \dots, v_N[/tex] cu erorile de măsură (așa cum am definit eroarea anterior și nu eroarea reală) [tex]\delta v_1, \delta v_2, \dots, \delta v_N[/tex].
Calculând media aritmetică a valorilor reale [tex]\overline{v}[/tex] putem spune că [tex]v_{real}\in \left [ \overline{v}-\delta \overline{v}, \overline{v}+\delta \overline{v} ][/tex] unde [tex]\delta \overline{v}=\frac {\sum_{i=1}^{N} \delta v_i} {N}[/tex]. Putem observa că prin mediere nu am crescut precizia măsurătorii, pentru claritate putem presupune că [tex]\delta v_1=\delta v_2=\dots=\delta v_N=\delta v[/tex] pentru că, de obicei, se folosesc aparate din aceeași clasă de precizie sau același aparat iar măsurătorile se fac în condiții similare, în condițiile acestea [tex]v_{real}\in \left [ \overline{v}-\delta v, \overline{v}+\delta v ][/tex]. Dacă nu am crescut precizia măsurătorii de ce se recomandă totuși să facem mai multe măsurători? Ne aducem aminte de incertitudine, prin mediarea valorilor masurate am reușit să reducem incertitudinea de la să zicem 0,1% la 0,001%, nici nu este nevoie de foarte multe măsurători ca să reducem incertitudinea de o sută de ori (nu mă intreba cum se determină, găsești sigur articole pe web).

Citat din: mircea_p din Februarie 02, 2011, 10:38:19 PM
In legatura cu pierderile prin convectie, un site NASA arata o imagine cu importanta diferitelor mecanisme de pierdere de caldura.
Dupa ei, 5% se pierde prin convectie si 17% prin radiatie.
Eu nu am mijloacele de a verifica asta, nici experimental nici teoretic, din pacate.
Și restul de pâna la 100% cum se pierde?
Oricum, faptul că se pierde mai puțină prin convecție nu înseamnă decât că se pierde suficient de multă prin radiație și nu mai apucă să se piardă suficientă prin convecție ceea ce înseamnă că gazele alea cu efect de seră nu prea-și fac treaba.
În realitate cam toată energia pierdută de Pământ se face până la urmă prin radiație pentru că atmosfera este înconjurată de vid destul de înaintat. Când m-am referit la convecție m-am referit la transportul energiei din păturile inferioare ale atmosferei către cele superioare, odată ajunsă în păturile superioare evident că energia se pierde prin radiație.

Citat din: Adi din Februarie 02, 2011, 10:41:08 PM
Citat din: mircea_p din Februarie 02, 2011, 10:27:04 PM
Citat din: tavy din Februarie 02, 2011, 08:31:17 PM
Poate distanța între Venus și Soare să aibă o importanță mai mare în faptul că temperatura pe Venus este mai mare decât pe Pământ?
Chiar este? Cum putem sti daca temperatura pe Venus e mai mare? Ce sens fizic are termenul "temperatura pe Venus"? Ori presupui ca intreaga planeta e la echilibru termic?
Haha, l-ai prins aici!
Ups. Nu știam că jucăm prinsea.
Serios vorbind, cred că și pe Venus se înregistrează temperaturi minime și maxime care pot fi comparate cu minimele și maximele de pe Pământ. Când mă refer că ,,temperatura pe Venus este mai mare decât pe Pământ" mă refer la comparații între aceste minime și maxime.
Titlu: Re: Măsurarea temperaturii atmosferice și punerea în evidență a schimbărilor climatice
Scris de: mircea_p din Februarie 02, 2011, 11:44:23 PM
Citat din: tavy din Februarie 02, 2011, 11:01:24 PM
O să încerc să fiu mai explicit.
Presupunem că masurăm mărimea fizică [tex]M[/tex] cu un aparat de măsură și obținem valoarea [tex]v[/tex]. În acest moment știm că valoarea mărimii [tex]M[/tex], [tex]v_{real}[/tex] se găsește în intervalul [tex]\left [ v-\delta v, v+\delta v \right ][/tex], unde [tex]\delta v[/tex] este dat de modul în care fac măsurătoarea, acest [tex]\delta v[/tex] îl numesc eu eroare în contextul dat.
Acum, noi știm că [tex]v_{real}[/tex] se găsește în intervalul [tex]\left [ v-\delta v, v+\delta v \right ][/tex] cu o anumită certitudine (probabilitate), cu alte cuvinte există o anumită încertitudine, incertitudine datorată factorilor aleatori și care poate fi estimată cu oarece precizie dar modul de estimare deja este alt subiect și prea complicat ca să-l explic aici (ne facem că nu observăm că am uitat cum se face).
Mergem mai departe și facem [tex]N[/tex] masurători ale mărimii fizice [tex]M[/tex] obținând valorile [tex]v_1, v_2, \dots, v_N[/tex] cu erorile de măsură (așa cum am definit eroarea anterior și nu eroarea reală) [tex]\delta v_1, \delta v_2, \dots, \delta v_N[/tex].
Calculând media aritmetică a valorilor reale [tex]\overline{v}[/tex] putem spune că [tex]v_{real}\in \left [ \overline{v}-\delta \overline{v}, \overline{v}+\delta \overline{v} ][/tex] unde [tex]\delta \overline{v}=\frac {\sum_{i=1}^{N} \delta v_i} {N}[/tex]. Putem observa că prin mediere nu am crescut precizia măsurătorii, pentru claritate putem presupune că [tex]\delta v_1=\delta v_2=\dots=\delta v_N=\delta v[/tex] pentru că, de obicei, se folosesc aparate din aceeași clasă de precizie sau același aparat iar măsurătorile se fac în condiții similare, în condițiile acestea [tex]v_{real}\in \left [ \overline{v}-\delta v, \overline{v}+\delta v ][/tex]. Dacă nu am crescut precizia măsurătorii de ce se recomandă totuși să facem mai multe măsurători? Ne aducem aminte de incertitudine, prin mediarea valorilor masurate am reușit să reducem incertitudinea de la să zicem 0,1% la 0,001%, nici nu este nevoie de foarte multe măsurători ca să reducem incertitudinea de o sută de ori (nu mă intreba cum se determină, găsești sigur articole pe web).

Era vorba de media valorilor masurate, nu a celor reale.
O valoare masurata se poate imagina ca
V_masurat= v_real+eroare.
Eroarea nu este o valoare fixa ci este o variabila aleatoare.
Cand facem media mai multor masuratori, avem
<V_masurat>= <v_real>+<eroare>
Cand eroarea e aleatoare e la fel de probabil sa fie pozitiva si negativa deci se poate observa si fara demonstratii statistice ca media erorii poate (si de obicei este) mai mica decat o anumita eroare individuala.
Asta e la inceputul oricarei carti de statistica a masuratorilor. Si poti sa te convingi de asta jucandu-te cu niste date simulate, in Excel de exemplu.
In practica metrologica se face diferenta intre precizie si acuratete in masuratori (iar tu ai introdus si incertitudinea, nu stiu la care din ele te referi). In termenii acestia, medierea valorilor poate creste "acuratetea".
Intervalul (+/-) 0.5 (sa zicem)  al unui aparat indica "precizia", cat de departate pot fi doua masuratori succesive ale unei marimi constante. Daca faci o singura masuratoare si ai ghinion, poti face o eroare de 0.5, adica esti la 0.5 unitati distanta fata de valoarea "reala". Daca faci mai multe masuratori (si nu ai eroare sistematica), media va fi mai aproape de valoarea reala decat 0.5.
Statistica iti poate chiar spune care este probabilitatea (nu certitudinea) de a fi la anumita distanta de valoarea "adevarata", in functie de numarul de masuratori si forma distribuitiei (care depinde de precizia instrumentului, printre altele).

Eu am vorbit de eroare, cum este de obicei numita in laboratoarele de fizica (calculul erorilor) si sper ca acum am aratat clar la ce m-am referit.


Titlu: Re: Măsurarea temperaturii atmosferice și punerea în evidență a schimbărilor climatice
Scris de: AlexandruLazar din Februarie 02, 2011, 11:56:07 PM
Exact, problema este cu siguranţă dată de faptul că terminologia folosită este, în stilul minunat al metrologiei, plină de capcane, gropi, catacombe, tranşee, şanţuri şi alte alea  ;D.

Să încercăm mai bine cu un exemplu practic. Să zicem că vrem să măsurăm tensiunea continuă furnizată de o baterie de 9V. Dacă scala voltmetrului are 100 de diviziuni şi capătul de scală e la 10V, ar trebui în principiu sa ajungem la 90 de diviziuni.

Incertitudinea inerentă adusă de voltmetru este de 100 mV, că atâta este între diviziuni. Dacă acul pică undeva între 90 şi 91 de diviziuni, nu se poate spune că tensiunea măsurată e de 9030 sau 9050 mV. Dacă însă repetăm măsurătoarea de foarte multe ori şi găsim ba 89, ba 90, ba 91, ba 90 şi un pic, ba 89 fără un pic, şi facem media, o să dea vreo 90 de diviziuni adică vreo 9000 mV.

Dacă în schimb voltmetrul este prost adus pe 0, astfel încât în loc de 90 de diviziuni, atunci când i se aplică 9V pe intrare, el arată 95, indiferent de câte ori repetăm măsurătoarea şi facem media, tot prost o să dea. Culmea, prin mediere o să ne apropiem de ce "vede" voltmetrul, dar nu şi de rezultat. Adică, prin mediere se poate creşte certitudinea dar nu precizia măsurării. E cam ca atunci când rezolvi o problemă. Dacă o rezolvi prost, şi repeţi aceleaşi calcule de 10 ori (la fel de prost), eşti sigur că ce a dat e bine, deşi în realitate n-ai fost deloc precis în rezolvare ;D.

Asta nu înseamnă că rezultatul obţinut prin medierea măsurătorilor nu este mai bun. Dacă se poate garanta precizia aparatelor (prin calibrare, folosirea unor referinţe verificate periodic ş.a.m.d.), medierea rezultatelor dă o măsurare mai certă -- adică în mod sigur, atâta vreme cât aparatele măsoară precis, rezultatul obţinut prin medierea suficientor rezultate este mai aproape de cel real decât rezultatul unei singure măsurători. Sigur, matematic ne putem imagina că nu e aşa -- adică putem să ne imaginăm că se aliniază planetele şi toate aparatele greşesc în plus, dar asta nu se întâmplă niciodată în practică.

Edit: mâine sau vineri dimineaţa când ajung la birou promit să mă uit în cărţoiul de metrologie legală şi să vă spun şi termenii exacţi pentru fiecare din drăciile astea pentru că, naiba sa le ia, e un capitol întreg numai despre ele şi nu le ştiu pe dinafară. Dealtfel treaba e suficient de încâlcită încât pe majoritatea foilor de lucru, îndrumarelor, fişelor de catalog şi în general orice altceva decât certificate legale şi alte hârţoage pe care inginerii le scriu ca să facă mişto de avocaţi, imediat lângă numele mărimii se trece şi formula de calcul ca să nu fie dubii despre ce crede fiecare că înseamnă eroarea relativă la domeniul de măsurare, eroarea raportată la domeniul de măsurare, incertitudinea de măsurare, incertitudinea erorii nu-ştiu-de-care şi aşa mai departe ;D. Metrologii sunt mai speciali într-adevăr, pentru ei eroare se cheamă cea introdusă de aparat pentru toate măsurătorile numai dacă e inerentă lui şi-i mereu aceeaşi, incertitudine e cea introdusă de aparat pentru toate măsurările dacă e aleatoare, dar diferenţa între un singur rezultat al unei măsurători şi valoarea reală se cheamă tot eroare ;D.

În cazul staţiilor meteo, dacă tot discutam de astea, avantajul e că totuşi calibrarea se poate face foarte uşor, aşa că de fapt problemele vin din incertitudine şi se rezolvă prin mediere.
Titlu: Re: Măsurarea temperaturii atmosferice și punerea în evidență a schimbărilor climatice
Scris de: tavy din Februarie 03, 2011, 12:06:48 AM
Citat din: mircea_p din Februarie 02, 2011, 11:44:23 PM
Citat din: tavy din Februarie 02, 2011, 11:01:24 PM
O să încerc să fiu mai explicit.
Presupunem că masurăm mărimea fizică [tex]M[/tex] cu un aparat de măsură și obținem valoarea [tex]v[/tex]. În acest moment știm că valoarea mărimii [tex]M[/tex], [tex]v_{real}[/tex] se găsește în intervalul [tex]\left [ v-\delta v, v+\delta v \right ][/tex], unde [tex]\delta v[/tex] este dat de modul în care fac măsurătoarea, acest [tex]\delta v[/tex] îl numesc eu eroare în contextul dat.
Acum, noi știm că [tex]v_{real}[/tex] se găsește în intervalul [tex]\left [ v-\delta v, v+\delta v \right ][/tex] cu o anumită certitudine (probabilitate), cu alte cuvinte există o anumită încertitudine, incertitudine datorată factorilor aleatori și care poate fi estimată cu oarece precizie dar modul de estimare deja este alt subiect și prea complicat ca să-l explic aici (ne facem că nu observăm că am uitat cum se face).
Mergem mai departe și facem [tex]N[/tex] masurători ale mărimii fizice [tex]M[/tex] obținând valorile [tex]v_1, v_2, \dots, v_N[/tex] cu erorile de măsură (așa cum am definit eroarea anterior și nu eroarea reală) [tex]\delta v_1, \delta v_2, \dots, \delta v_N[/tex].
Calculând media aritmetică a valorilor reale [tex]\overline{v}[/tex] putem spune că [tex]v_{real}\in \left [ \overline{v}-\delta \overline{v}, \overline{v}+\delta \overline{v} ][/tex] unde [tex]\delta \overline{v}=\frac {\sum_{i=1}^{N} \delta v_i} {N}[/tex]. Putem observa că prin mediere nu am crescut precizia măsurătorii, pentru claritate putem presupune că [tex]\delta v_1=\delta v_2=\dots=\delta v_N=\delta v[/tex] pentru că, de obicei, se folosesc aparate din aceeași clasă de precizie sau același aparat iar măsurătorile se fac în condiții similare, în condițiile acestea [tex]v_{real}\in \left [ \overline{v}-\delta v, \overline{v}+\delta v ][/tex]. Dacă nu am crescut precizia măsurătorii de ce se recomandă totuși să facem mai multe măsurători? Ne aducem aminte de incertitudine, prin mediarea valorilor masurate am reușit să reducem incertitudinea de la să zicem 0,1% la 0,001%, nici nu este nevoie de foarte multe măsurători ca să reducem incertitudinea de o sută de ori (nu mă intreba cum se determină, găsești sigur articole pe web).

Era vorba de media valorilor masurate, nu a celor reale.
O valoare masurata se poate imagina ca
V_masurat= v_real+eroare.
Eroarea nu este o valoare fixa ci este o variabila aleatoare.
Cand facem media mai multor masuratori, avem
<V_masurat>= <v_real>+<eroare>
Cand eroarea e aleatoare e la fel de probabil sa fie pozitiva si negativa deci se poate observa si fara demonstratii statistice ca media erorii poate (si de obicei este) mai mica decat o anumita eroare individuala.
Asta e la inceputul oricarei carti de statistica a masuratorilor. Si poti sa te convingi de asta jucandu-te cu niste date simulate, in Excel de exemplu.
In practica metrologica se face diferenta intre precizie si acuratete in masuratori (iar tu ai introdus si incertitudinea, nu stiu la care din ele te referi). In termenii acestia, medierea valorilor poate creste "acuratetea".
Intervalul (+/-) 0.5 (sa zicem)  al unui aparat indica "precizia", cat de departate pot fi doua masuratori succesive ale unei marimi constante. Daca faci o singura masuratoare si ai ghinion, poti face o eroare de 0.5, adica esti la 0.5 unitati distanta fata de valoarea "reala". Daca faci mai multe masuratori (si nu ai eroare sistematica), media va fi mai aproape de valoarea reala decat 0.5.
Statistica iti poate chiar spune care este probabilitatea (nu certitudinea) de a fi la anumita distanta de valoarea "adevarata", in functie de numarul de masuratori si forma distribuitiei (care depinde de precizia instrumentului, printre altele).

Eu am vorbit de eroare, cum este de obicei numita in laboratoarele de fizica (calculul erorilor) si sper ca acum am aratat clar la ce m-am referit.
Deci ceea ce numeam eu eroare tu numești precizie, nu este foarte important atâta timp cât am spus la ce mă refer prin termenul eroare în contextul respectiv.
Tu susții că dacă ai un aparat de măsură cu precizia (folosesc temenul tău acum) de 0,5 unități dacă fac mai multe măsurători pot determina valoarea mărimii masurate cu o precizie mai bună de 0,5. Poți să-mi spui și mie câte măsurători să fac cu rigla pentru a avea o precizie de 0,1mm? Asta ca să nu mai am nevoie de șubler. Eventual, dacă nu te deranjază, spune-mi și câte măsurători trebuie să fac cu rigla ca să înlocuiesc micrometrul (precizie 0,01mm).
Titlu: Re: Măsurarea temperaturii atmosferice și punerea în evidență a schimbărilor climatice
Scris de: mircea_p din Februarie 03, 2011, 12:35:44 AM
Hai ca esti carcotas.
Ori te faci ca nu intelegi ori chiar nu intelegi. In ambele cazuri mie mi se pare neinteresanta continuarea discutiei.
Titlu: Re: Măsurarea temperaturii atmosferice și punerea în evidență a schimbărilor climatice
Scris de: tavy din Februarie 11, 2011, 12:47:12 PM
Citat din: Adi din Ianuarie 31, 2011, 10:13:47 PM
Nu inteleg care parte din asta nu e clar. Pana la urma e irelavant daca media temperaturilor creste sau nu, cat conteaza energia disponibila in atmosfera si in oceane. Si acesta energie creste incontinuu, iar dupa ce se va topi gheta de la poli, energia se va duce direct in cresterea de temperatura in mod mai accelerat.
În legătură cu energia disponibilă în atmosferă, ne-am aștepta ca o creștere a ei să ducă la mai multe furtuni și de întensitate mai mare, uite ce am găsit eu: Global Tropical Cyclone Activity (http://www.coaps.fsu.edu/~maue/tropical/index.html).
A se remarca că în articol nu se folosesc mărimi pseudoștiințifice de genul medierii mărimilor fizice intensive.
(http://www.coaps.fsu.edu/~maue/tropical/global_running_ace.jpg) (http://www.coaps.fsu.edu/~maue/tropical/global_running_ace.jpg)

(http://www.coaps.fsu.edu/~maue/tropical/graphics/global_running_freq_12.jpg) (http://www.coaps.fsu.edu/~maue/tropical/graphics/global_running_freq_12.jpg)

Dacă am avea într-adevăr de-a face cu o ,,încălzire globală" care să conteze cât de cât ne-am aștepta ca numărul uraganelor și al furtunilor să crească sau măcar energia lor să crească. În realitate se poate observa o ușoară scădere atât a frecvenței furtunilor/uraganelor cât și a energiei lor.
Cu toate acestea climatoologi care ne prognozează încălziri catastrofale nu tin cont de aceste aspecte ci iau în considerare doar fenomene cum ar fi topirea banchizei de la polul nord, fenomene care nu nici o relație directă de cauzalitate cu o încălzire/răcire globală.
Trebuie să spun că nu susțin că energia cumulată a furtunilor/uraganelor ar avea vreo relevanță în ce privește încălzirea globală, nu știu doar intuiesc că ar putea exista o legătură.

Nu știu cum a determinat autorul articolului energia total acumulată de uragane, pare ceva destul de dificil, dar cel puțin nu este teoretic imposibil de determinat cum este o așa zisă ,,temperatură globală", energia fiind o mărime intesivă și în consecință aditivă.
În ce privește frecvența furtunilor/uraganelor este deasemenea o mărime fizică care poate fi ușor determinată în contrast cu ,,temperatura medie" care nu este o mărime fizică,
Titlu: Re: Măsurarea temperaturii atmosferice și punerea în evidență a schimbărilor climatice
Scris de: AlexandruLazar din Februarie 11, 2011, 05:22:20 PM
CitatDacă am avea într-adevăr de-a face cu o ,,încălzire globală" care să conteze cât de cât ne-am aștepta ca numărul uraganelor și al furtunilor să crească sau măcar energia lor să crească.

Nu neapărat -- de fapt frecvența și intensitatea furtunilor nu este cauzată de tendințe "globale" ci de diferențele de presiune și temperatură.
Titlu: Re: Măsurarea temperaturii atmosferice și punerea în evidență a schimbărilor climatice
Scris de: tavy din Februarie 11, 2011, 05:44:45 PM
Citat din: AlexandruLazar din Februarie 11, 2011, 05:22:20 PM
Nu neapărat -- de fapt frecvența și intensitatea furtunilor nu este cauzată de tendințe "globale" ci de diferențele de presiune și temperatură.
Iar diferențele de presiune și temperatură înseamnă acumulare de energie, energie care în cazul pământului vine de la soare.
Am pornit de la ideea că încălzire ar însemna că atmosfera primește mai multă căldură decât cedează, consider că este singurul mod de a spune, fără a apela la mărimi nefizice, specifice numerologiei, cum ar fi ,,temperatura medie", că avem de-a face cu o încălzire.

Așa cum am mai spus:
Citat din: tavy din Februarie 11, 2011, 12:47:12 PM
Trebuie să spun că nu susțin că energia cumulată a furtunilor/uraganelor ar avea vreo relevanță în ce privește încălzirea globală, nu știu doar intuiesc că ar putea exista o legătură.
Am explicat acum de ce intuiesc că ar putea avea o legătură. Intuiția mea cel puțin se bazează pe ceva spre deosebire de ceea ce susțin climatologii care se bazează pe mărimi definite arbitrar de ei (vezi ,,temperatura medie globala"=,,media ponderată a temperaturilor măsurate") fără nici o legătură cu fizica.
Energia furtunilor sau numărul lor pot fi adunate, mediate, etc. temperaturile nu pot fi adunate și în consecință nici mediate.
Titlu: Re: Măsurarea temperaturii atmosferice și punerea în evidență a schimbărilor climatice
Scris de: tavy din Februarie 11, 2011, 06:48:25 PM
Se pare că s-au prins și alții că se cheltuiesc prea mulți bani pe studiul schimbărilor climatice fără rezultate.

Reps Posey, Adams and Bishop Join Colleagues in Calling on House Leaders to Reprioritize NASA for Human Space Flight Missions, Drop Climate Change (http://www.comspacewatch.com/news/viewpr.rss.html?pid=32693)
Citat
In Fiscal Year 2010, NASA spent over 7.5% --over a billion dollars-- of its budget on studying global warming/climate change. The bulk of the funds NASA received in the stimulus went toward climate change studies. Excessive growth of climate change research has not been limited to NASA. Overall, the government spent over $8.7 billion across 16 Agencies and Departments throughout the federal government on these efforts in FY 2010 alone.

NASA climate programs being eyed for the budget axe (http://wattsupwiththat.wordpress.com/2011/02/09/nasa-climate-programs-being-eyed-for-the-budget-axe/)
Citat
NASA spent over a billion dollars last year on climate change studies...which would you rather have? Pronouncements about death trains, expert testimony for climate vandals, failed predictions, failed models, and a questionable GISTEMP dataset, or a continued manned spaceflight program?
Titlu: Re: Măsurarea temperaturii atmosferice și punerea în evidență a schimbărilor climatice
Scris de: Adi din Februarie 11, 2011, 07:04:12 PM
Se stie ca reprezentatii partidului republican din SUA sunt pro-afaceri, chiar cu riscul de a afecta grav mediul prin poluare (afacerile cu petrol, cu armata, etc). Ei sunt de aceea mai opusi ideii de incalzire globala si schimbari climatice, caci ei ar vrea sa extra petrol si din zonele Arctice. Asadar sunt sceptic. Dar e dreptul lor sa faca intrebari catre NASA. Sa vedem care va fi decizia intregului parlament american.
Titlu: Răspuns: Re: Măsurarea temperaturii atmosferice și punerea în evidență a schimbărilor climatice
Scris de: Stark din Octombrie 29, 2014, 03:53:22 PM
[]
Titlu: Răspuns: Răspuns: Re: Măsurarea temperaturii atmosferice și punerea în evidență a schimbărilor clima
Scris de: tavy din Octombrie 29, 2014, 04:07:45 PM
Citat din: Stark din Octombrie 29, 2014, 03:53:22 PM
Citat din: tavy din Februarie 03, 2011, 12:06:48 AM
Deci ceea ce numeam eu eroare tu numești precizie, nu este foarte important atâta timp cât am spus la ce mă refer prin termenul eroare în contextul respectiv.
Tu susții că dacă ai un aparat de măsură cu precizia (folosesc temenul tău acum) de 0,5 unități dacă fac mai multe măsurători pot determina valoarea mărimii masurate cu o precizie mai bună de 0,5. Poți să-mi spui și mie câte măsurători să fac cu rigla pentru a avea o precizie de 0,1mm? Asta ca să nu mai am nevoie de șubler. Eventual, dacă nu te deranjază,spune-mi și câte măsurători trebuie să fac cu rigla ca să înlocuiesc micrometrul (precizie 0,01mm).

Una singura!

Daca vreau sa masor grosimea unei foi de hartie folosind doar o rigla gradata (1mm precizie), atunci masor grosimea unui top de o mie foi, si impart valoarea masurata la 1000.
Precizia masuratorii este de 1mm/1000= 0.001mm. Neah, ca a batut  pana si micrometrul. Si asta doar cu o singura masuratoare. ;D
Numai că nu ai nu ai măsurat grosimea unei foi de hârtie, ai măsurat grosimea unui top și ai determinat grosimea unei foi presupunând că toate foile din top au aceeași grosime. Dacă nu faci presupunerea că toate foile au aceeași grosime, calculul erorilor se schimbă.
Titlu: Răspuns: Răspuns: Re: Măsurarea temperaturii atmosferice și punerea în evidență a schimbărilor clima
Scris de: Stark din Octombrie 29, 2014, 04:13:03 PM
[]
Titlu: Răspuns: Măsurarea temperaturii atmosferice și punerea în evidență a schimbărilor climatice
Scris de: Electron din Octombrie 29, 2014, 05:11:12 PM
Trecand peste parseci si nevoia de a avea 1000 de foi "la fel" cu cea de interes, cum evaluam incertitudinea masurarii, presupunand o distributie gaussiana a grosimilor? Presupun ca intervine cumva si "largimea clopotului" (scuzati limbajul), parametru care intra la ipoteze.

Care ar fi metoda de calcul a incertitudinii masuratorii pentru grosimea foii buclucase, in functie de parametrii distributiei ?


e-
Titlu: Răspuns: Măsurarea temperaturii atmosferice și punerea în evidență a schimbărilor climatice
Scris de: Stark din Octombrie 29, 2014, 05:22:27 PM
[]
Titlu: Răspuns: Măsurarea temperaturii atmosferice și punerea în evidență a schimbărilor climatice
Scris de: Electron din Octombrie 29, 2014, 05:49:43 PM
Da, eu l-am folosit, pentru ca asa am intalnit termenul in acest context.

Cand am invatat eu despre masuratori, orice masuratoare avea o incertitudine, o "furculita", adica nu e suficient sa dai valoarea masurata "fix la fix cu 100 de zecimale" ci trebuie sa precizezi un interval min/max unde se situeaza valoarea in cauza.

Concret, nu are sens sa spui ca ai masurat o lungime de 13,5 cm, ci eventual 13,5 +/- 0,1 cm. Aici incertitudinea este de 0,2 cm.

In principal incertitudinea e legata de precizia aparatului (gradatia minima de pe "rigla" aparatului), dar aceasta este afectata si de alti factori, printre care formula finala de calcul (daca de fapt masori mai multi termeni/factori din formula pe care ii aduni, imparti etc), sau ipoteze legate de proprietatile obiectului masurat (cum e cazul celor 1000 de foi unde presupui ca toate sunt "la fel").

Sunt curios, daca se ia ca ipoteza o distributie gaussiana a grosimii celor 1000 de foi, cum afecteaza asta incertitudinea, pentru ca intuitiv impartirea cu 1000 merge doar pentru foi "fix identice".


e-
Titlu: Răspuns: Măsurarea temperaturii atmosferice și punerea în evidență a schimbărilor climatice
Scris de: Stark din Octombrie 29, 2014, 06:19:10 PM
[]
Titlu: Răspuns: Răspuns: Măsurarea temperaturii atmosferice și punerea în evidență a schimbărilor climatice
Scris de: Electron din Octombrie 30, 2014, 10:04:06 AM
Ok, merci pentru raspunsul detaliat.


Citat din: Stark din Octombrie 29, 2014, 06:19:10 PMDe ce nu folosesti termenul de abatere standard, si continui sa-i zici incertitudine?
Probabil pentru ca am fost obisnuit cu el si imi suna mai bine termenul francofon decat cel anglofon. Daca termenii sunt clar definiti, confuziile se pot evita.

CitatDaca vrei si sigma distributiei gaussiene a topului de hartie... acu vino si tu cu o idee (masuratori independente cu topuri cu nr de foi diferit nu stiu daca merge). 
Intrebarea mea era pentru cazul in care valoarea lui sigma distributiei gaussiene este data prin ipoteza, adica atunci cand vrem sa evaluam abaterea standard a masuratorii in functie de valoarea lui sigma.

Daca ceea ce vrem este sa determinam pe sigma distributiei gaussiene a grosimii foilor, atunci trebuie sa avem masuratori cat mai precise pentru fiecare foaie individuala, iar asta presupune sa avem o precizie a masuratorii fiecarei foi individuale notabil mai buna decat "largimea clopotului", ceea ce e practic invers decat situatia in care neglijam "largimea clopotului" (o cosideram foarte mica) si masuram 1000 de foi deodata ca sa estimam grosimea uneia.

e-
Titlu: Răspuns: Răspuns: Măsurarea temperaturii atmosferice și punerea în evidență a schimbărilor climatice
Scris de: Stark din Octombrie 30, 2014, 10:43:21 AM
[]
Titlu: Răspuns: Răspuns: Răspuns: Măsurarea temperaturii atmosferice și punerea în evidență a schimbărilor
Scris de: Electron din Octombrie 30, 2014, 11:29:54 AM
Citat din: Stark din Octombrie 30, 2014, 10:43:21 AM
Daca eu compar nota unui elev cu nota medie a clasei, vorbesc de o deviatie de la medie si nu de o eroare.
Iar deviatiile de la medie la nivelul clasei sunt caracterizate de un indicator statistic caruia i-as zice mai degraba abatere standard, decat eroare sau incertitudine.
Nici eu nu as folosi teremenul de incertitudine in acest context. Eu asociez acest termen cu masuratorile fizice. Compararea notei unui elev cu media clasei nu este o masuratoare pentru mine. (Daca este o masuratoare in context larg, educational, atunci e greseala mea ca fac aceasta falsa distinctie).

CitatFolosirea unui termen nepotrivit poate fi o sursa de confuzie, de acord?
De acord.

Pentru a evita confuziile, dat fiind ca lumea provine din medii si experiente personale diferite, deci termenii diferiti sunt inerenti, trebuie avut rabdare sa comparam definitiile respective. Cine se oftica si refuza sa dea definitiile pe care le foloseste (sau si mai rau, pretinde ca toata lumea intelege acelasi lucru ca si el fara sa dea vreo definitie), acela trebuie sa-si reconsidere atitudinea daca vrea sa discute serios.

CitatIn alt exemplu, cand eu am fost interpelat in discutia cu masuratori si medii de temperaturi mi
s-a vorbit de "incertitudine". Nu e de mirare ca mi-a fost greu sa inteleg ca interlocutorul se referea doar la erori de masura, cand in realitate marul discordiei era masurarea unei marimi aleatoare, si tocmai ca estimatorii distributiei ei de valori trebuia sa intre in "ecuatie".
Aici probabil e vina mea. Eu m-am referit cu incertitudinea doar la operatia de masurare a grosimii foii, nu la alte discutii (iar asta cu foaia e un off-topic de la discutia de baza). Daca precizam asta poate era mai clar.

CitatOK! Daca sigma este dat o sa revin, dar cred ca din ce am scris poti face si tu un calcul independent.
Din cate vad eu, ai dat deja raspunsul:

Citat din: Stark din Octombrie 29, 2014, 06:19:10 PM
Acu... pentru abaterea standard stii ca sunt doua surse de fluctuatie: una inerenta aparatului de masura si alta de la marimea aleatoare insasi.
Daca variantza unei sume este suma variantzelor (variantza este patratul abaterii standard) (sau alternativ via teorema de propagare a erorilor)
abaterea standard estimata mai sus e

[tex]
\sigma = \sqrt{\sigma_{aparat}^2+ \sigma_{natura}^2}
[/tex]


e-
Titlu: Răspuns: Răspuns: Răspuns: Măsurarea temperaturii atmosferice și punerea în evidență a schimbărilor
Scris de: Stark din Octombrie 30, 2014, 11:44:22 AM
[]
Titlu: Răspuns: Măsurarea temperaturii atmosferice și punerea în evidență a schi
Scris de: Electron din Octombrie 30, 2014, 12:09:41 PM
Citat din: Stark din Octombrie 30, 2014, 11:44:22 AM
Chiar vreau sa stiu un studiu de caz: cum e posibil sa stiu [tex]\sigma_{natura}[/tex], dar nu stiu grosimea medie a unei foi, si din acest motiv trebuie sa o masor...)
De exemplu putem sa ne bazam pe procesul de fabricatie care asigura ca sigmanatura e de un anumit ordin, fara sa stim care e grosimea "nominala" la care a fost calibrata presa, adica ce grosime ar trebui sa aiba foile.

Dupa cum ai vazut, interesul era si calitativ, nu doar cantitativ. Adica, daca rezulta ca pentru sigmanatura suficient de mic, procesul de stacking il poate elimina, atunci avem o informatie practica: Daca putem presupune ca foile sunt suficient de uniforme, putem sa le masuram cu o precizie imbunatatita, chiar daca avem la dispozitie o rigla si nu un micrometru. :)

Daca ipoteza despre sigmanatura e gresita, masuratoarea noastra lasa de dorit. E bine totusi sa avem ipotezele explicit, pentru ca din ipoteze implicite rezulta tot felul de nazbatii. ;)

e-
Titlu: Răspuns: Măsurarea temperaturii atmosferice și punerea în evidență a schi
Scris de: Stark din Octombrie 30, 2014, 12:20:34 PM
[]

Titlu: Răspuns: Răspuns: Măsurarea temperaturii atmosferice și punerea în evidență a schi
Scris de: Electron din Octombrie 30, 2014, 12:38:37 PM
Citat din: Stark din Octombrie 30, 2014, 12:20:34 PM
Informatia legata de sigma este eliminata prin stacking oricum, indiferent de cat de mare sau mic este [tex]\sigma[/tex].
Important e sa ai un nr reprezentativ de probe (numarul de foi sa fie mare N =100, sau 1000 etc.)
Ok.

De retinut atunci ca, daca sigma acela e prea mare, degeaba il eliminam prin stacking pentru a masura media, ca tot nu avem informatii prea precise despre foile individuale.

CitatDaca faci masuratori pentru un top de hartie din care vrei sa extragi informatia ptr sigma... forget it. (Merge, poate, numai daca N este mai degraba mic 50, sa zicem 100,  in nici un caz 1000;  si daca sigma ar fi de ordinul 1mm/10 sa zicem, si asta pentru un numar oarecare de topuri...)
De acord.

Citat
Citat
E bine totusi sa avem ipotezele explicit, pentru ca din ipoteze implicite rezulta tot felul de nazbatii
De ce imi faci precizarea asta?
Nu iti este adresata tie neaparat, ci este o observatie in general. Am intalnit foarte adesea indivizi care nu doar ca folosesc ipoteze implicite, dar devin irascibili cand vrei sa le explicitezi. Nu lua chiar totul personal. :)


e-
Titlu: Răspuns: Răspuns: Măsurarea temperaturii atmosferice și punerea în evidență a schi
Scris de: Stark din Octombrie 30, 2014, 12:49:05 PM
[]
Titlu: Re: Măsurarea temperaturii atmosferice și punerea în evidență a schimbărilor cli
Scris de: Electron din Octombrie 31, 2014, 10:04:43 AM
Citat din: Stark din Octombrie 30, 2014, 12:49:05 PM
Iti dau o distributie gaussiana pentr valorile unei marimi aleatoare. Distributia e definita de 2 parametrii: media [tex]\mu[/tex] si abaterea standard [tex]\sigma[/tex].
Si iti precizez urmatoarele valori numerice:

[tex]
\mu = 0.35 u
\sigma = 10.3 u
[/tex]

unde u = unitate (m, Kg, K etc);

Intrebare: ti se pare ciudat ca ti-am dat media cu doua zecimale exacte desi abaterea standard este de ordinul a 10 unitati?
Nu mi se pare ciudat. Sigma poate sa aiba orice ordin, ca doar e vorba de abaterea standard a marimii (cum vine ea de la mama natura).

Media se obtine din masuratori si calcule, iar daca masuratorile sunt suficient de multe si de exacte (de la aparatul de masura citire), poate sa aiba si mai multe zecimale.

Oricum, mie mi se pare ca se amesteca doua lucruri in asemenea intrebari: incertitudinile masuratorilor (inerente aparatului de masura) care apar si daca masuram exact acelasi "obiect" de mai multe ori, cu variatia valorilor aferente mai multor "obiecte" asemanatoare, dar diferite, care dau distributia "naturala", adica marimile aleatoare.

Eu am intervenit in discutie atunci cand a fost vorba de o masuratoare (a unui "obiect") si am intrebat ce impact are distribuita "naturala" in cazul acesteia. (Si apropos, nu am intentia sa prelungesc off-topicul).


e-
Titlu: Răspuns: Re: Măsurarea temperaturii atmosferice și punerea în evidență a schimbărilor cli
Scris de: Stark din Octombrie 31, 2014, 10:44:50 AM
[]
Titlu: Răspuns: Răspuns: Re: Măsurarea temperaturii atmosferice și punerea în evidență a schimbărilor cli
Scris de: Electron din Octombrie 31, 2014, 11:54:28 AM
Citat din: Stark din Octombrie 31, 2014, 10:44:50 AM
Tu nu poti insinua acum ca in exemplu ala cu topul de hartie as fi facut exact greseala pe care tocmai am taxat-o (ignorand caracterul aleator al marimii masurate insasi, si prin analogie, ignorand inconstient eventuala distributie de grosimi a foilor din top).
Imi acorzi prea mult credit. Nu am intentia sa insinuez nimic de acest fel.

Eu am pus o intrebare pentru a afla ceva pe o tema cu care am avut tangente cam de multicel: incertitudinea masuratorilor.


e-